SlideShare une entreprise Scribd logo
1  sur  21
O.O. Bohomolets National Medical University

                   CHAIR OF GENERAL SURGERY No. 1



                                  Approved by methodical meeting of the chair
                                  "__"___2007, minutes No.___

                                  Prof., MD, PhD. O.I. Dronov




           METHODICAL GUIDES FOR PRACTICAL LESSONS
                  Materials for teachers and students

                                 Subject:
Peritonitis: Inflammation of the peritoneal cavity. Classifications, clinics,
                         diagnosis, prophylaxis




                                        Course 3
                                        Faculties 2 and 3, faculty for
                                        educating the doctors for Ukrainian
                                        Army, faculty of medical psychology
                                        Duration: 4 academic hours
                                        Prepared by assistant
                                        M.D. S.V. Zemskov



                                 Kyiv 2007
2


I.     Priorities of the subject:

        The concept of the relative and absolute indications for a surgery exists in the surgical
practice. The absolute indication is the patient's condition which could not be corrected by the
conservative means. Peritonitis or he threat of peritonitis is the most prevalent absolute
indication for the urgent abdominal surgery. Therefore, the knowledge of the clinical features of
peritonitis and the major principles of its treatment is of vital importance for the clinicians.

II.    The aims of training:

А-1. Scope of the knowledge:
- Anatomy and histology of the peritoneum and the greater omentum;
- Resorptive and exudative properties of the peritoneum;
- Biological and chemical characteristics of the secretions of the glands comprising the
    digestive system and other biological fluids of the body capable of contacting the
    peritoneum;
- Features of the local and general inflammatory response of the peritoneum;
- Clinical symptoms and syndromes peculiar to peritonitis


А-2. Student should be acquainted with the following subjects:
- Mechanisms of the development of localized and general peritonitis;
- Extent of clinical manifestations depending on the etiology of peritonitis;
- Pathogenesis of complications due to peritonitis;
- Hazards of peritonitis and importance of the timely adequate treatment.


 А-3. Scope for practical skills:
 - Competent anamnesis for the assessment of the causes of peritonitis;
 - Use of physical examination techniques and assessment of pathognomonic symptoms of
    peritonitis;
 - Timely recognition of peritonitis and use of the require complex of the diagnostic
    procedures for confirmation / refusal of peritonitis diagnosis.

 А-4. List of practical technique to be trained:
 - Detection of Schotkin-Blumberg symptom;
 - Interpretation of the survey X-ray film of peritoneal cavity
 - Interpretation of the ultrasonographic data.

III.   Training and educational objectives:
1. To demonstrate the importance of the subject for the timely and adequate treatment of
   peritonitis.
2. To demand the adherence to the principles of the medical ethics and deontology.
3. To demonstrate the importance of the knowledge and skills in the field employing the
   practical examples.
3


IV.     Interdisciplinary integration:

Subject for study and Scope of knowledge                       Scope of skills
corresponding chair
                          Pathological states resulting in     Detection and interpretation of
General surgery with care
                          complications manifesting as         pathognomonic symptoms of
for patients
                          peritonitis                          peritonitis
                                                               Interpretation of peritonitis
                              Anatomical structure and
                                                               probability in case of injury of
Human anatomy                 relation of the peritoneal
                                                               the specified organ located in
                              organs to the peritoneum
                                                               peritoneal cavity
                              Structure and embryogenesis
Histology and embryology
                              of peritoneum
                                                               Differential prescription of
                              Microflora of the specific
Microbiology                                                   antibacterial agents according to
                              divisions of the intestines
                                                               the etiology of peritonitis
                              Exudative and resorptive         Indications for drainage / lavage
Physiology
                              functions of peritoneum          of the peritoneal cavity

V. Content of training

                                         INTRODUCTION

        The peritoneum is a two-layered serous membrane that lines the abdominal cavity
covering the inside walls of the abdomen (parietal peritoneum) and the abdominal organs
(visceral peritoneum). Passing from the abdominal walls to the intra-abdominal organs and from
one organ to another one, the peritoneum generated the folds and the mesenteries bordering the
spaces, the sinuses, and the recesses.
        The peritoneal cavity is limited by the diaphragm from above, the pelvic diaphragm and
the ilia from below, then backbone and the muscles of the back posteriorly, the rectus muscles of
abdomen anteriorly, and the internal oblique and transverse muscles of abdomen anteriorly and
laterally.
        The peritoneum is a semipermeable actively functioning membrane possessing several
functions such as exudative-and-resorptive function and the barrier function (due to migrating
and resident macrophages, immunoglobulins, and non-specific factors.
        Histologically, the peritoneum comprises six layers: mesothelium, basal membrane and
four layers of elastic and collagenous fibers. On average, the thickness of the serous membrane is
about 0.2 mm.
        The total area of the peritoneal cover amounts to 17000-20400 square centimeters, which
practically equals to the area of the skin surface.
4

          The exudative areas of the peritoneum are represented for the most part by the serous
cover of the intestines. The most intensive exudation is in the duodenal area with the gradual
decrease in caudal direction.
          The most intensive resorption is inherent to the peritoneum of the diaphragm, the greater
omentum, the ileum, and the cecum. The amount of the fluid absorbed by peritoneum within one
hour makes up to 8% of body mass.
          The greater omentum represents the duplicature of the peritoneum containing the fat with
well-developed blood and lymphatic vessels providing for the protective function circumscribing
the foci of the inflammation by means of the fibrin.
          The peritoneum is supplied by the blood from the basins of the vessels feeding the
corresponding organs. The outflow of the venous blood is provided mainly through the portal
system and partly through the caval system. The most intensive outflow of the lymph occurs
from the surface of the greater omentum and the diaphragm.
          The visceral peritoneum has the vegetative innervation (parasympathetic and
sympathetic) and practically is devoid of the somatic innervation. This is the reason why the
visceral pains due to the irritation of the visceral peritoneum are not localized. The most
sensitive are so called reflex areas of the root of mesentery, the areas of the celiac trunk, the
pancreas, the ileocecal angle, and Douglas pouch. The parietal peritoneum is devoid of the
somatic innervation. This fact explains the absence of the protective tension of the muscles of the
anterior abdominal wall in the case of the inflammations in the small pelvis.


                                 PERITONITIS CLASSIFICATION
                               (after Yu.M. Lopukhin and V.S. Saveliev)
І.        According to the clinical course: acute and chronic.
ІІ.       According to the route of penetration of bacteria into the peritoneal cavity:
А. Primary peritonitis when the infection spreads by hematogenic or lymphogenic route or via
the fallopian tubes.
B. Secondary peritonitis when the infection penetrates due to the acute surgical conditions or the
injury of the peritoneal organs.


          1. Infectious and inflammatory peritonitis results from the following diseases of the
peritoneal organs:
      - acute appendicitis,
      - acute cholecystitis,
      - acute ileus,
5

       - acute pancreatitis,
       - thromboembolia of the mesenterial blood vessels,
       - diverticulitis,
       - intestinal tumors,
       - gynecological conditions.
           2. Perforative peritonitis results from the perforations due to:
       - duodenal and gastric ulcers;
       - intestinal ulcerations associated with typhus, dysentery, tuberculosis, cancer, stress, etc.)
       - decubitus ulcer in the obturation ileus;
       - strangulation groove in strangulated intestinal obstruction in the foreign bodies of
       gastrointestinal tract;
       - intestinal necrosis in the strangulated hernia or thromboembolia of the mesenterial blood
       vessels.
           3. Traumatic peritonitis develops in the open and closed abdominal traumas both with
accompanying damage of the abdominal organs and without such damage.
           4. Postoperative peritonitis results from:
       - the failure of the sutures of anastomosis after the abdominal surgery;
       - the infection of the peritoneal cavity in the course of the surgery;
       - the defects of the ligatures applied to the large areas of the omentum and the mesentery
       followed by the necrosis of the tissues distal to the ligature
       - the mechanical damage of the peritoneum: drying or the hemorrhage into the free
       abdominal cavity without the reliable hemostasis.


ІІІ.       According to the microbiological features
1.         Bacterial peritonitis: non-specific caused by microflora of the gastrointestinal tract and
specific caused by microflora, which is not associated with the gastrointestinal tract such as
gonococci (Neіsserіa gonorrhoeae), pneumococci (Streptococcus pneumonіaе), hemolytic
streptococci (Streptococcus pyogenes, Streptococcus vіrіdans), micobacteria of tuberculosis
(Mycobacterіum tuberculosіs).
2.         Aseptical peritonitis develops because of the exposure of peritoneum to the toxic and
enzymatic agents of non-infectious origin (the blood, the bile, the gastric juice, the chylous fluid,
the pancreatic juice, the urine, the aseptic necrosis if the internal organs).
3.         Specific forms of peritonitis:
Carcinomatous.
Parasitic.
6

Rheumatoid.
Granulomatous.
ІV.    According to the character of the peritoneal exudate.
"      Serous
"      Fibrinous
"      Purulent
"      Hemorrhagic
V.     According to the character of the lesions of the peritoneal surface
1.     Depending on the circumscription, the following forms of peritonitis may be delineated:
The circumscribed peritonitis may be represented as an abscess or infiltrate (for example,
paravesical abscess or pericholecystitis as the complication of the gangrenous cholecystitis)
Non-circumscribed peritonitis lacks the strict boundaries and the tendencies towards the
circumscription.
2.     Depending on the extent of the spread, the following forms of peritonitis may be
delineated :
Localized: confined only to one anatomical division of the abdominal cavity.
Diffuse: extended to 2-5 anatomical divisions of the abdominal cavity.
Generalized: total involvement of the peritoneum (6 and more divisions of the abdominal cavity.
VІ.    According to the phases of the development.
1.     Reactive phase (first 24 h, 12 h for perforative peritonitis)
2.     Toxic (24-72 h, 12-24 h for perforative peritonitis)
3.     Terminal (more than 72 h, more than 24 h for perforative peritonitis).


                      Definition of peritonitis as the surgical pathology

Peritonitis is defined as the acute inflammation of the peritoneum. Peritonitis represents one of
the most severe complication accompanying the diseases and the injuries of the intra-abdominal
organs and may be delineated as the separate nosology with the characteristic clinical pattern and
the complex of the severe pathophysiological reactions with the impairment of the whole
homeostasis systems. Referring to the peritonitis and its surgical treatment, one bears in mind
acute, secondary, non-specific peritonitis being the cause of complications of about 15-20 % of
all the acute pathologies of the abdominal organs.
       Among the forms of the acute peritonitis, the acute appendicitis with the incidence of
30-65% ranks the first followed by the perforated ulcer (7-15 %), the acute cholecystitis (10-12
%), the gynecological pathologies (3-12 %), the ileus (3-5 %), the pancreatitis (1 %), and
postoperative peritonitis (1 %).
7

The chronic peritonitis may develop in tuberculosis, mycosis, carcinomatosis, also in ascitic
conditions and in the presence of the numerous syphilitic gummas.
The aseptic peritonitis develops
   -   Upon irritation of the peritoneum with various chemical agents (iodine, ethanol, non-
       isotonic solutions, some antiseptics, enzymes);
   -   Upon irritation of the peritoneum with the aggressive biological fluids (urine, bile,
       pancreatic or gastric juice, the content of the hydatid cyst),
   -    Upon the extensive ligation of the tissues.
All the forms indicated above make up to less than 1 % of all peritonitis cases, the rest being
attributed to the acute secondary peritonitis.


                      CLASSIFICATION OF SURGICAL PERITONITIS

        The general classification is rather bulky for everyday use. That is why in so called
working classifications the cause of the peritonitis is given (the perforated ulcer or acute
gangrenous perforated appendicitis) with accompanying morphological description (diffuse,
fibrinous-purulent peritonitis). The words such as "secondary" and "infectious non-specific" are
usually being avoided).
        In final diagnosis, the complications resulting from the infectious (septic) processes and
postoperative complications are referred to (if any).
        Circumscription and extent of spreading: The redundant terms (circumscribed – local or
non-circumscribed – diffuse) should not be used together. The area of the lesion should be
strictly indicated.
        Phases of the development: We use the pathogenetic classification after K. Simonian:
        Reactive phase: The manifestations of the peritonitis are caused by the activated defense
systems such as the massive release of the kinins in the response to the primary infectious
aggression. Therefore, when the source of infection is eliminated, in reactive phase the radical
surgery is possible with anastomoses being applied to the abdominal organs. In the inflammatory
process, the reactive phase lasts up to 24 hours (in perforative peritonitis up to 12 hours). Upon
the source of the peritonitis is eliminated (the perforated ulcer, for example) in the early
(reactive) phase, the peritonitis does not progress as usual and the lethality is rather low.
        Toxic phase is characterized by the alterations in the functions of all systems of the body
because of the syndrome of the systemic inflammatory response. The toxic phase is evident in
12-24 hours after the onset of the disease in perforative processes and 24-48 hours in the
inflammatory processes. The lethality is 20% and higher.
8

       Terminal phase is characterized by the development of the polyorgan insufficiency
syndrome representing the depletion of the functional reserves of the major systems of the body.
The toxic phase is evident in 24-36 hours in the perforative peritonitis and in 48-72 hours in the
inflammatory processes.
       The lethality in the patients with polyorgan insufficiency syndrome is about 90%.

Complications
     The local and general complications may be delineated.
       The local complications comprise pylephlebitis, the multiple abscesses of the liver.
       The general complications comprise the liver insufficiency, the toxic encephalopathy, the
renal failure, the cardiovascular failure.
       The predominant part of the peritoneum covers the intestines. It is naturally, therefore,
that the toxic products flowing out with blood or lymph impair microcirculation in the intestinal
wall and neuromuscular transmission resulting in decreasing and arresting the peristalsis
(intestinal paresis). The intestinal content is a good milieu for the development of bacteria
including the anaerobic ones since the aerobic bacteria realize the available oxygen. The arrest of
the intestinal passage results in the increasing microbial content in the middle and upper thirds of
the small intestine where the bacteria are normally absent.
       The serous edema extends to the whole intestinal wall. The mucosa loses the barrier
function. Ads a result, the bacteria and their metabolites are absorbed into the lymph and the
blood circulation not only from the peritoneal cavity but from the intestine as well. Therefore,
since the onset of the intestinal paresis in peritonitis, the intoxication and dehydration of the
body progress rapidly.
       The increase in the toxic load to the liver results in then rapid depletion of its detoxifying
function and other metabolic functions as well. The penetration of the bacteria and toxins into
the caval system results in extra loading to the lungs resulting in the development of the
inflammation in the lung tissue. As a consequence, the respiratory function is impaired with the
development of the respiratory distress syndrome resulting finally in the pneumonia.


Microbiological features of peritonitis
       As a rule, the exudate is infected with variety of bacterial species inhabiting the cavities
and the surfaces of the human body.
       The most prevalent batteries are the Escherichia coli, Enterococcus, and Proteus sp. The
putrefactive odor and the sordid of the exudate suggest that the predominant microflora relates to
the facultative anaerobic bacteria (so-called non-clostridial anaerobic microflora).
9

       Earlier, gas chromatography was used as the express method for detecting the bacteria
comprising the exudates based on the analysis of the specific bacterial metabolites – volatile
fatty acids (propionic, butyric, valeric). The up-to-date techniques of detection and identification
are based on the immunocytochemistry and polymerase chain reaction.
       Upon the prolonged treatment, the nosocomial infection caused by penicillinase-
producing cocci or Pseudomonas aerugіnosa may be evident.
       The results of the commonly used bacteriologic techniques are evident in three days.
Therefore, the initial antibiotic therapy should be based on the previous experience in the field.
The following findings should be taken into account:
       – Gram-positive cocci are the major bacterial species in the case of the localization of the
primary focus in the upper division of the gastrointestinal tract (the stomach, the duodenum, the
bile ducts, the pancreas, upper third of the small intestine). The probability of the involvement of
the anaerobic bacteria is rather low (10-15%) and depends entirely on the duration of the
process.
       – Gram-negative bacilli and non-clostridial anaerobic bacteria prevail in the perforation
of the small intestine and the appendix as well as in the cases of peritonitis with the
accompanying ileus.
       – Escherichia coli ranks among the first in the development of the peritonitis when the
perforation is not available.

                      SYMPTOMATICS AND DIAGNOSIS OF PERITONITIS
       The diagnosis of the peritonitis is based on the complex assessment of the anamnesis, the
local symptoms, the presence of the inflammation and intoxication, and the specific
complications.
       Complaints: The steady blunt pain throughout the abdomen increasing upon the
movements and the breathing. The irradiation into the shoulder is possible upon the irritation of
the diaphragm. The general sickness is evident with frequent nausea and stool retention.


       Anamnesis
       The time passed since the onset of the symptoms should be clarified. The dynamics of the
character and the localization of the pain, the dynamics of the toxic manifestations are followed.
In most cases, the signs of the previous disease (appendicitis, cholecystitis, acute condition of the
ulcer) resulting in peritonitis are evident. Frequently, in the setting of some improvement of the
general state and decreasing of the abdominal pains, the sudden increase of the pain occurs
10

which becomes more extensive. Since this moment, the general state of the patient deteriorates,
the dryness in mouth and the thirst appear, the palpitation increases.


          General examination
          As usual, the patient lies in supine position with the knees bent. The attempts to change
the pose or to rise fail due to the increasing pain. When the patient sits, the attempt to lie down
results in increasing abdominal pain and the pain irradiating to the shoulders due to the irritation
of the diaphragmatic nerve. The patient is forced to take up the previous sitting position (the
tumbler toy symptom).
          The patient speaks quietly and does not shout claiming the attention. Such a behavior of
the patient especially the breathing accompanied by the quiet moaning is a sign of particular
concern for the doctor.
          All the components of the infectious process and the intoxication such as the temperature,
the tachycardia, the rate, and the deepness of breathing, the changes in blood pressure, the
dryness of the tongue and the internal buccal surface should be taken into account. The loss of
consciousness should be of special concern signifying the severe intoxication.
          The tachycardia with heart rate of 100-120 beats per minutes is evident with normal or
decreased blood pressure and the increased respiration rate (20-24 breathings per minute).
          The toxic encephalopathy may be manifested both as the inhibition and the excitation in
the form of delirium.
          The paleness of the skin and cutis marmorata reflects the deep disorder of
microcirculation.


          Specific examination
          The abdomen is usually symmetrical, somehow distended, is not involved in breathing
movements. By palpation, the diffuse pain, the tension of the abdomen, and Mendel and
Schotkin-Blumberg symptoms are revealed. The peristalsis sounds are usually weakened or
absent.
          The presence of the free gas should be checked by the disappearance of the liver dullness.
The presence of the free liquid is proved by the dullness of the percussion sound in the lateral
divisions disappearing upon turning the patient to the side.
          In rectal and vaginal examination, the overhang of the fornices and the tenderness due to
the accumulation of the inflammatory exudates are frequently evident.
          In general blood analysis, the pronounced leukocytosis and the shift towards the
immature forms are present.
11

        Biochemically, normo- or hyperglycemia, the moderate increase of urea, creatinine, the
alanine aminotransferase, aspartate aminotransferase, alkaline phosphatase activities are
detected; the syndrome of the disseminated intravascular blood coagulation may develop.


       Laboratory examination
       The moderate leukocytosis (14-20 х 109/L) with the shift of the formula to the left,
lympho- and monopenia, aeosinophilia, thrombocytopenia are evident. For the objective
assessment of the intoxication extent, Calf-Calif leukocytic index of intoxication is calciulated
with its value of 4 characteristic of the reactive phase, the value of 8 characteristic of the toxic
phase, and the value of 12-18 characteristic of the terminal phase.
       The increase of the hematocrit, the increased content of urea and creatinin, the increased
activity of transaminases, the elevation of bilirubin, glucose, lactate, the alteration of the
parameters of coagulation system, the changes of acid-alkaline state, the increase in the blood
partial pressure of CO2 and the decrease in the blood partial pressure of O2 are the signs of the
impaired functions of various organs and systems.


       Additional examinations
       The presence of the free fluid in the abdominal cavity may be confirmed sonographically.
X-ray examination may be useful for detecting free gases, the signs of ileus (Kloiber bowls), the
high position of the diaphragmatic cupolas and the limitation of their mobility due to the tension
of the abdominal muscles (the muscular defense). In the severe cases, the respiratory distress
syndrome develops.
         In doubtful cases, laparoscopy may be performed for clarifying the source and the
extent of the source of the peritonitis. The diagnosis of peritonitis may be confirmed by
laparocentesis by the presence of the specific exudate.
       The manifestations of the peritonitis in reactive and terminal phases are quite different. In
the reactive phase, the pain syndrome prevails. The area of the utmost tenderness corresponds to
the primary inflammatory focus. The rigidity of the abdomen, due to contraction of the muscles
of the abdominal wall is a characteristic feature. The tense abdomen looks somehow tucked and
boat-like. This sigh is of peculiar importance for the peritonitis with perforation of the hollow
organs. Mendel and Schotkin-Blumberg symptoms are revealed. The peristalsis sounds are
usually weakened or absent. The overt signs of dehydration are absent. The tachycardia may
reach 90-100 beats per minute. The respiration rate is not increased. Several elements of the
excitation phase of the shock such as the elevated blood pressure may also be evident. The
12

neutrophilic leukocytosis 12-18 х 109/L, lympho- and monopenia, aeosinophilia, may be present.
The biochemical parameters are usually within the normal limits.
       In the terminal phase the diagnosis of the peritonitis may be rather difficult if the
anamnestic details are not available. The vomiting with the congestive content takes place
several times a day. The stool is scarce, frequently only after the enema. The stool has the
putrefactive odor.
       The objective state: Adynamia, the general inhibition, the acute dehydration, the hollow
eyes, the sharpened features of the face, the superficial frequent respiration, frequently with the
quiet moaning. The tongue is dry and difficult to hang out; the internal buccal surfaces are also
dry. The abdomen is inflated, weakly painful. Mendel and Schotkin-Blumberg symptoms are
rather dubious. The percutory sound is not uniform. The symptoms of the deathly silence may be
evident when the respiratory and the vascular noises are heard instead of the intestinal
movements. The splash sound may be detected when the abdominal wall is stroke by the hand.
       The multiple Kloiber bowls are evident in survey X-ray pictures. The tachycardia is
above 120 beats per minute. The blood pressure is decreased, especially after the transportation
of the patient. The maintenance of the normal blood pressure requires the intensive infusion. The
breathing rate is about 30 per minute. The oxygen saturation of the blood is diminished. The
shadows of various sides in the lungs are characteristic of the respiratory distress syndrome.
Hydrothorax may be developed. In blood, normocytosis, sometimes leukopenia are detected with
the sharp shift to the left (towards the immature elements), sometimes plasma cells are detected.


                         GENERAL PRINCIPLES OF TREATMENT

1.     The early elimination of the infection source.
2.     The decrease of the possible intraoperative contamination.
3.     The treatment of the residual infection and the prevention of further infection of the
abdominal cavity.
4.     The maintenance of vitally important functions prior to and after the surgery, incuding
the techniques requiring the artificial organs.

                                  METHOD OF TREATMENT
А. Surgical
Laparotomy, early elimination or isolation of the peritonitis source.
Intra- and postoperative sanitization of the abdominal cavity.
Decompression of the small intestine.
13

B. General
Massive antibioticotherapy.
Medicamentous correction of hemostasis shifts.
Stimulation or temporary substitution of the major detoxifying systems of the body by means of
extracorporeal hemocorrection.


         PREOPERATIVE PREPARATION. It is impossible to correct the hemostasis
impairment prior to the surgery. Nevertheless, it is advantageous to delay the surgery for 2-3
hours for directed preparation. It is sufficient to stabilize arterial pressure and central venous
pressure and to reach the diuresis at 25 mL/h. The total preoperative infusion volume is 1.5-2.0 L
within two hours. In advanced cases when the hemodynamic imbalance is pronounced (the loss
of fluid exceeding 10% of body mass), the infusion volume increases up to 3-4 L within 2-3
hours.
         The catheterization of the central (for example subclavian) vein is always performed
providing for high infusion flow rate and the control of the central venous pressure. The
catheterization of the urinary bladder is expedient providing for measuring the diuresis as the
objective criterion of the infusion therapy efficacy.
         PREPARATION OF GASTROINTESTINAL TRACT. The stomach is emptied with the
aid of the probe. In the advanced cases, the permanent presence of the probe in the stomach is
expedient pre-operatively, intraoperatively and postoperatively for specified time until the
recovery of gastric motor activity.
         ANESTHESIA. The multicomponent anesthesia with the artificial lung ventilation is
used in surgery due to peritonitis.


                       MAJOR OPERATIVE STAGES IN PERITONITIS
1.       Surgical access.
2.       Possible Novocain blockade of the reflexogenic areas.
3.       Elimination or reliable isolation of septic focus.
4.       Sanitization of the abdominal cavity.
5.       Decompression of the intestine.
6.       Draining of the abdominal cavity.
7.       Suturing of the laparotomic wound.


         1. Surgical access. The optimal access to all divisions of the abdominal cavity is provided
by the midline laparotomy. Depending on the localization of the septic focus, the wound may be
14

extended in superior or inferior direction. When the diffuse peritonitis is revealed in the course
of the surgery started from another access, it is worth to perform the midline laparotomy.
        2. Novocain blockade of the reflexogenic areas is performed by administering 0.5%
solution of Novocain (up to 100.0 mL) into the area of the celiac trunk, the root of the
mesocolon, transverse mesocolon, the mesentery of the small intestine, and sigmoid mesocolon.
Such blockade eliminates the reflex vascular spasm providing for earlier recovery of peristalsis.
In addition, such blockade allows diminishing the amount of the narcotic substances used as the
analgesics.
        3. Elimination or reliable isolation of septic focus. In the reactive phase, the radical
operations (resection of the stomach, hemicolectomy) are possible since the risk of the
anastomosis failure is rather low. In the toxic and terminal phases the scope of the surgery should
be minimal (appendectomy, the suturing of the perforative hole, the resection of the necrotic area
of the gastrointestinal tract with entero- or colostoma, the isolation of the septic focus from the
free abdominal cavity. The reconstructive surgery should be postponed until more favorable
conditions.
        4. Sanitization of the abdominal cavity. The washing of the abdominal cavity allows for
decreasing the bacterial count in the exudates below the critical level (105 in 1 mL) facilitating
the elimination of the infection. The dense deposits of fibrin are not removed because of the risk
of deserozation. The removal of exudate by means of the wiping with the gauze pieces is
unacceptable due to the risk of the injury of the serous membrane. The washing fluid must be
isotonic. The use of antibiotics is meaningless because the short-term contact with the
peritoneum is not sufficient for their effects. Most antiseptics are cytotoxic; this fact restricts
their use in the cases under consideration. With this aim, one may use sodium chloride solution
upon electrochemical activation (0.05% sodium hypochlorate) containing activated chlorine and
oxygen. This solution is particularly indicated in then cases when anaerobic bacteria are present.
Several clinics also use the ozonized solutions.
        5. Decompression of the intestine. In the toxic and terminal phases when the intestinal
paresis is of particular clinical importance, nasogastrointestinal intubation is performed. The
polyvinylchloride tube is used with this aim. The intubation is extended distally from Treitz
ligament. When necessary, the colon is intubated through the anus. Rarely, gastro-, jejuno- or
appendicostoma is required for passing the probe. Postoperatively, the probe-assisted correction
of the enteral content is performed comprising the decompression, the intestinal lavage, the
enterosorption, and early enteral feeding. All these measures are useful for decreasing the
permeability of the intestinal walls for the bacteria and the toxins facilitating early recovery of
the functional activity of the gastrointestinal tract.
15

          6. Draining of the abdominal cavity is performed by the tubes made of polyvinylchloride
or rubber. Multichanneled polyvinylchloride drainages are the good choice for this procedure.
One channel is used for supplying the antiseptic, while another one is used for the aspiration of
the exudate.
          7. Suturing of the laparotomic wound may be performed with the draining of the
subcutaneous fat.
          The treatment of the residual infection, namely the techniques of the draining and the
exudate removal, depends on the techniques used for the final stages of the surgery.


          Methods of suturing laparotomic wound:
          1. The tight suturing without the drainage may be employed only in the case of the local,
non-circumscribed peritonitis serous peritonitis with the low level of the bacterial load and low
risk of abscesses and infiltrates. In these cases, the self-defense of the body and the use of
antibiotics may be sufficient for curbing the infection.
          2. The suturing with the passive drainages. The drainages are also used for the local
administration of antibiotics.
          3. The suturing with the drainages for the flow and fractional lavage. The technique is not
employed in wide scale due to the difficulties in correcting the shifts in protein and electrolytic
balance. Moreover, the efficacy of the technique lowers in 12-24 hours since the beginning of the
lavage.
          4. Semi-closed technique with approaching of the wound edges provides for the
drainages being installed in the posterior abdominal wall for the dorsoventral washing with the
aspiration of the outflowing fluid via the midline wound.
          5. The approaching of the wound edges with the specific appliances used for the repeated
revisions and sanitizations. We use the term "scheduled laparosanitization". The indication for
such technique is the pronounced adhesion process in the severe forms of the fibrinopurulent
peritonitis with sub- and decompensation of the vitally important functions. The number of the
revisions is from 2-3 to 7-8 with 1248-hours interval.
          6. The open technique (laparostomy after N. Macokh or Steinberg-Miculich) with the
exudate outflow through the wound covered by the tampons with the ointment is used in the case
of the multiple non-shaped intestinal fistulas. The doctor may observe the condition of the
intestinal loops adjacent to the wound in every change of the tampons.
16

                                    GENERAL TREATMENT

Antibacterial therapy
The most adequate schedule of the empirical antibacterial therapy before the microbiological
verification of the causative agent and the assessment of its sensitivity to antibiotics is a
combination of the synthetic penicillins (Ampicillin) or cephalosporins with aminoglycosid
(Gentamicin or Vancomicin) and Metronidasol. Such a combination seems to be effective
against the broad spectrum of the possible causative agents of the peritonitis.
       When the data of bacteriological analysis are obtained, the corresponding antibiotics or
their combination are prescribed.
       The routes of the administration:
       1) Local (intraperitoneal) through irrigators and drainages (the double function of
draining)
       2) General
       a) intravenous (systemic)
       b) intramuscular (only after the recovery of microcirculation)
       c) intra-arterial (into aorta, celiac trunk, mesenteric or omental artery)
       d) intraportal (through the recanalized umbilical vein of the round ligament of the liver
(regional way)
       e) endolymphatic
                 – antegradely: through microsurgically catheterized peripheral lymphastic vessel
in the back of the foot, the depulpated inguinal lymph node
                 – retrogradely: through thoracic lymphatic duct
                 – lymphotropically: through the lymphatic net of the shin or retroperitoneal space.


       Immunotherapy
       The following preparations improving the immune reactivity of the body may be used:
immunoglobulins (antistaphylococcus gamma-globulin), the leukocytic mass, antistaphylococcus
plasma. Interleukin-2, α- and γ-interferons, interferon inducers (cycloferon) also may be
employed as the modern drugs improving the immune reactivity. Meanwhile, the use of
Pyrogenal, Decaris (Levamisol), Prodigiosan, Thymalin and other agents "stimulating the
reduced immunity" seems to de contraindicated.
17

        Postoperative correction therapy
        For the adequate anesthetization, besides the narcotic analgesics, the prolonged epidural
analgesia with the local analgesics, acupuncture analgesia, and electroanalgesia are employed.
        The balanced infusion therapy is performed. The total daily volume of the infused fluid
comprises the physiological daily requirement (1500 mL/m2), the deficit of the liquid, and the
physiological expenses (vomiting, drainage, increased sweating, and hyperventilation).
        The preventing and the treatment of multiple organ failure syndrome:
        The pathogenetic bases of the multiple organ failure syndrome is hypoxia and cell
hypotrophy due to the impaired respiration and hemodynamics.
        The following measures should be undertaken to prevent and to treat multiple organ
failure syndrome:
        – Elimination of the infectious and toxic focus.
        – Elimination of the toxins by the techniques of the efferent therapy.
        – Maintenance of the adequate lung ventilation and gas exchange (sometimes long=-term
artificial lung ventilation).
        – Stabilization of blood circulation with restoration of the blood circulation volume, the
support of heart function. Normalization of microcirculation in organs and tissues.
        – Correction of protein, electrolyte, and acid-base balance of the blood.
        – Parenteral feeding
        Recovery of functions of gastrointestinal tract
        The most effective means for recovering the functions of gastrointestinal tract consists in
the intestinal decompression by transnasal probe followed by the intestinal lavage.
        For the normalization of the nervous regulation and the recovery of the tonus of intestinal
musculature, the correction of the protein and electrolyte balance is performed. Then
anticholinesterase drugs (Proserin, Ubretid), Metaclopramide and ganglioblockers (Dimecolin,
Benzohexonium) may be used.
        The forced diuresis, hemodialysis, plasmapheresis, hemofiltration through the porcine
organs, the artificial lung ventilation, hyperbaric oxygenation are indicated in case of the
multiple organ failure syndrome. The hyperbaric oxygenation is capable for arresting all types of
hypoxia in peritonitis, facilitates the decrease in the bacterial load of the peritoneum, stimulates
motor and evacuatory function of the intestine.
        For preventing endotoxicemia, the surgical methods and antibacterial therapy are to be
used for eliminating the residual infection.
        Hemosorption, lymphosorption, plasmapheresis and other detoxifying techniques are to
be regarded only as the supplementary methods in the complex therapy of peritonitis.
18

         VI. Plan and arrangement of training

         6.1 Duration of classes: 4 academic hours
         6.2. Stages of classes:

N      Content and     program    of Educa-          Methods       of Supporting            Time
       teaching                      tional          teaching     and materials             in
                                     objectives      control                                min
                                     within
                                     learning
                                     levels
1      2                             3               4                5                     6
І      Preparatory step                                               Log-book for
1      Organization of classes                                        entering the          5
2      Formulation of the teaching                                    learning progress
       goals                                                          of students
3      Preparatory stage (control of
       the initial level of knowledge                                 Tables, slides        20
       and skills):                     α-ІІ         Tests α-ІІ       Tables, slides
       а) major concepts on the                                                             20
       anatomy and physiology of        α-ІІ         Tests α-ІІ       Tables, slides
       peritoneum;                      α-ІІ         Tests α-ІІ                             15
       б) infectious process;                        Typical check
       в) approaches for the surgical                situations
       treatment;
ІІ     Principal step                                                 Equipment and         120
1.     Training the professional                                      facilities
       skills and knowledge                          Training
2.     Examination of the patients      α-ІІІ
       with peritonitis                              Non-typical
3.     Differential; diagnosis of                    check
       peritonitis. Diagnostical and                 situations
       clinical features.               α-ІІІ
ІІІ    Final step                                    Individual       Control questions ,
1.     Control and correction of the                 control of       tasks.                50
       attained level of professional   α-ІІ         students'        Advice for
       skills                                        knowledge        homework with the     5
2.     Summary of classes                                             literature
       (theoretical, practical,         α-ІІІ
       organizational)                                                                      5
3.     Home tasks


VII Supporting materials required for teaching

      Materials for the control of initial level of knowledge and skills:
      А. Control questions:
      1. Anatomical relations between the peritoneum and the organs of the abdominal cavity
      2. Histological structure and innervation of the peritoneum.
19

   3.  Physiology of the peritoneum: resorptive and exudative functions.
   4.  Definition of peritonitis.
   5.  Classification of peritonitis according to the clinical course.
   6.  Classification of peritonitis according to the character of penetration of bacteria into the
       abdominal cavity.
   7. Infectious inflammatory peritonitis.
   8. Perforative peritonitis.
   9. Traumatic peritonitis.
   10. Postoperative peritonitis.
   11. Classification of peritonitis according to the microbiological features.
   12. Bacterial peritonitis.
   13. Aseptic peritonitis.
   14. Specific forms of peritonitis.
   15. Classification of peritonitis according to the character of the peritoneal exudate.
   16. Classification of peritonitis according to the character of lesions on the surface of the
       peritoneum.
   17. Circumscribed peritonitis.
   18. Non-circumscribed peritonitis.
   19. Classification of peritonitis according to the extent of spreading.
   20. Localized peritonitis.
   21. Generalized (diffuse) peritonitis.
   22. General (total) peritonitis.
   23. Phases of the development of peritonitis.
   24. Reactive phase of peritonitis.
   25. Toxic phase of peritonitis.
   26. Terminal phase of peritonitis.
   27. Complications of peritonitis.
   28. Microbiological features of peritonitis.
   29. Patients' complaints and the anamnestic data in peritonitis.
   30. General and specialized examination of the patients with peritonitis.
   31. Laboratory examinations
   32. Supplementary examinations.
   33. General principles of treating peritonitis.
   34. Method for treating peritonitis.
   35. Preoperative preparation.
   36. Preparation of gastrointestinal tract.
   37. Anesthesia.
   38. The major steps of the surgical treatment of peritonitis.
   39. Draining of the abdominal cavity and suturing of the laparotomic wound in peritonitis.
   40. The general treatment of peritonitis.

   TESTS

І. The most pronounced exudative activity is peculiar to:
    A. peritoneum of ileum;
    B. peritoneum of duodenum;
    C. peritoneum is devoid of exudative activity;
    D. peritoneum of diaphragm;
    E. peritoneum of sigmoid colon.
20


ІІ. The most pronounced resorptive activity is peculiar to:
           A. peritoneum of jejunum;
           B. peritoneum of duodenum;
           C. peritoneum is devoid of resorptive activity;
           D. peritoneum of diaphragm and greater omentum;
           E. peritoneum of sigmoid colon.

ІІІ. The peritoneum covers (completely or partially) all the organs of the abdominal cavity except
for:
     A. pancreas;
     B. kidneys;
     C. urinary bladder;
     D. omentum;
     E. in the abdominal cavity all the organs without exception are covered by the peritoneum.

   IV.     Select the wrong statement – peritonitis may be:
           A. primary;
           B. secondary;
           C. perforative;
           D. traumatic;
           E. all the statements are correct.

   V.      Infectious and inflammatory peritonitis is not a consequence of:
           A. acute appendicitis;
           B. acute cholecystitis;
           C. acute ileus;
           D. traumatic injuries of parenchymatous organs;
           E. gynecological pathology.

   VI.     Which of the form in the list below is not regarded as the specific form of peritonitis:
   A. infectious;
   B. parasitic;
   C. rheumatoid;
   D. granulomatous;
   E. carcinomatous.

   VII.    Which of the pathologies from the list below do not belong to the systemic
           complications of peritonitis:
           A. pylephlebitis;
           B. syndrome of the disseminated intravascular blood coagulation;
           C. toxic encephalopathy;
           D. renal failure;
           E. cardiovascular failure.

   VIII.   Select the wrong statement – the following symptom does not belong to the
           pathognomonic symptoms of peritonitis:
           A. Mussi-Heorhievskyi symptom;
           B. Ortner symptom;
           C. Karavaiev symptom;
           D. Schotkin-Blumberg symptom;
           E. Pasternatsky symptom.
21


   IX.    The stage which does not comprises the major stages of the surgical treatment of
          peritonitis:
          A. operative access;
          B. elimination or reliable isolation of the septic focus;
          C. sanitization of the abdominal cavity;
          D. draining of the abdominal cavity;
          E. antibioticotherapy.

   Х. Select the wrong statement: Paresis of the intestine…
   A. ...accompanies the development of peritonitis;
   B. …is a pathogenetic factor contributing to the severity of the clinical course in peritonitis;
   C. …in peritonitis is subjected to the conservative correction;
   D. … is not observed in all cases of the local form of peritonitis;
   E. …is in most cases an indication for intestinal intubation.

   The correct answers:
   1–A            4–E             7–A            10 – C
   2–D            5–E             8–A
   3–E            6–D             9–D

REFERENCES
1. S.V. Petrov. «General surgery». Peter. 2005 (in Russian).
4. Yu.M. Lopukhin, V.S. Saveliev «Surgery» Moscow. 1997 (in Russian).
5. C.M. Townsend. Textbook of Surgery. The biological basis of modern surgical practice.
    2001.

Contenu connexe

Tendances

Tendances (20)

Benign breast disease
Benign breast diseaseBenign breast disease
Benign breast disease
 
PERIANAL ABSCESS & ISCHIORECTAL ABSCESS
PERIANAL ABSCESS & ISCHIORECTAL ABSCESSPERIANAL ABSCESS & ISCHIORECTAL ABSCESS
PERIANAL ABSCESS & ISCHIORECTAL ABSCESS
 
Acute appendicitis
Acute appendicitisAcute appendicitis
Acute appendicitis
 
Discuss the sysytemic disturbances of high intestinal fistula
Discuss the sysytemic disturbances of high intestinal   fistulaDiscuss the sysytemic disturbances of high intestinal   fistula
Discuss the sysytemic disturbances of high intestinal fistula
 
Intra abdominal abscess
Intra abdominal abscessIntra abdominal abscess
Intra abdominal abscess
 
Hydrocele
HydroceleHydrocele
Hydrocele
 
Right hemicolectomy
Right hemicolectomyRight hemicolectomy
Right hemicolectomy
 
Gastric Outlet Obstruction (GOO)
Gastric Outlet Obstruction (GOO)Gastric Outlet Obstruction (GOO)
Gastric Outlet Obstruction (GOO)
 
Suprapubic cystostomy
Suprapubic cystostomySuprapubic cystostomy
Suprapubic cystostomy
 
Peritonitis
PeritonitisPeritonitis
Peritonitis
 
Medical student surgery osce
Medical student surgery osceMedical student surgery osce
Medical student surgery osce
 
Stoma
StomaStoma
Stoma
 
Congenital hypertrophic pyloric stenosis
Congenital hypertrophic pyloric stenosisCongenital hypertrophic pyloric stenosis
Congenital hypertrophic pyloric stenosis
 
Approach to a patient with breast lump
Approach to a patient with breast lumpApproach to a patient with breast lump
Approach to a patient with breast lump
 
Basic surgical skills
Basic surgical skillsBasic surgical skills
Basic surgical skills
 
Peritonitis
PeritonitisPeritonitis
Peritonitis
 
Surgical site infections: Latest Approach on management.
Surgical site infections: Latest Approach on management.Surgical site infections: Latest Approach on management.
Surgical site infections: Latest Approach on management.
 
Day care surgery by manjusb
Day care surgery by manjusbDay care surgery by manjusb
Day care surgery by manjusb
 
Gallstone ileus
Gallstone ileusGallstone ileus
Gallstone ileus
 
Surgical Jaundice
Surgical JaundiceSurgical Jaundice
Surgical Jaundice
 

En vedette

Bohomolets Oncology Practical Methodical #1
Bohomolets Oncology Practical Methodical #1Bohomolets Oncology Practical Methodical #1
Bohomolets Oncology Practical Methodical #1Dr. Rubz
 
Learning outcome in Pediatric
Learning outcome in PediatricLearning outcome in Pediatric
Learning outcome in PediatricDr. Rubz
 
Febrile summary
Febrile summaryFebrile summary
Febrile summaryDr. Rubz
 
Rubzzzz's Neurology Case Hx 4th year
Rubzzzz's Neurology Case Hx 4th yearRubzzzz's Neurology Case Hx 4th year
Rubzzzz's Neurology Case Hx 4th yearDr. Rubz
 
Guides on GI system
Guides on GI systemGuides on GI system
Guides on GI systemDr. Rubz
 
Cardiac failure ( long case approach ) summary
Cardiac failure ( long case approach ) summaryCardiac failure ( long case approach ) summary
Cardiac failure ( long case approach ) summaryDr. Rubz
 
Rubzzzz's Nervous system on Hx taking
Rubzzzz's Nervous system on Hx takingRubzzzz's Nervous system on Hx taking
Rubzzzz's Nervous system on Hx takingDr. Rubz
 
Guides on Gastroenterology
Guides on GastroenterologyGuides on Gastroenterology
Guides on GastroenterologyDr. Rubz
 
Cpg child immunisation
Cpg child immunisationCpg child immunisation
Cpg child immunisationDr. Rubz
 
Short case approach to speech analysis summary
Short case approach to speech analysis summaryShort case approach to speech analysis summary
Short case approach to speech analysis summaryDr. Rubz
 
Short case approach to cushing summary
Short case approach to cushing summaryShort case approach to cushing summary
Short case approach to cushing summaryDr. Rubz
 
Guides on Heart Disease
Guides on Heart DiseaseGuides on Heart Disease
Guides on Heart DiseaseDr. Rubz
 
O&g examination
O&g examinationO&g examination
O&g examinationDr. Rubz
 
Guidelines of Epilepsy by NICE
Guidelines of Epilepsy by NICEGuidelines of Epilepsy by NICE
Guidelines of Epilepsy by NICEDr. Rubz
 
Short case approach to parkinson's dz summary
Short case approach to parkinson's dz summaryShort case approach to parkinson's dz summary
Short case approach to parkinson's dz summaryDr. Rubz
 
Short case approach to acromegaly summary
Short case approach to acromegaly summaryShort case approach to acromegaly summary
Short case approach to acromegaly summaryDr. Rubz
 
Gina asthma management
Gina asthma managementGina asthma management
Gina asthma managementDr. Rubz
 
HIV/AIDS data Hub Asia Pacific -Malaysia 2014
HIV/AIDS data Hub Asia Pacific -Malaysia  2014HIV/AIDS data Hub Asia Pacific -Malaysia  2014
HIV/AIDS data Hub Asia Pacific -Malaysia 2014Dr. Rubz
 
Chronic renal failure concise long case approach & crf with fluid overload m...
Chronic renal failure concise long case approach  & crf with fluid overload m...Chronic renal failure concise long case approach  & crf with fluid overload m...
Chronic renal failure concise long case approach & crf with fluid overload m...Dr. Rubz
 
Neurological examination summary
Neurological examination summaryNeurological examination summary
Neurological examination summaryDr. Rubz
 

En vedette (20)

Bohomolets Oncology Practical Methodical #1
Bohomolets Oncology Practical Methodical #1Bohomolets Oncology Practical Methodical #1
Bohomolets Oncology Practical Methodical #1
 
Learning outcome in Pediatric
Learning outcome in PediatricLearning outcome in Pediatric
Learning outcome in Pediatric
 
Febrile summary
Febrile summaryFebrile summary
Febrile summary
 
Rubzzzz's Neurology Case Hx 4th year
Rubzzzz's Neurology Case Hx 4th yearRubzzzz's Neurology Case Hx 4th year
Rubzzzz's Neurology Case Hx 4th year
 
Guides on GI system
Guides on GI systemGuides on GI system
Guides on GI system
 
Cardiac failure ( long case approach ) summary
Cardiac failure ( long case approach ) summaryCardiac failure ( long case approach ) summary
Cardiac failure ( long case approach ) summary
 
Rubzzzz's Nervous system on Hx taking
Rubzzzz's Nervous system on Hx takingRubzzzz's Nervous system on Hx taking
Rubzzzz's Nervous system on Hx taking
 
Guides on Gastroenterology
Guides on GastroenterologyGuides on Gastroenterology
Guides on Gastroenterology
 
Cpg child immunisation
Cpg child immunisationCpg child immunisation
Cpg child immunisation
 
Short case approach to speech analysis summary
Short case approach to speech analysis summaryShort case approach to speech analysis summary
Short case approach to speech analysis summary
 
Short case approach to cushing summary
Short case approach to cushing summaryShort case approach to cushing summary
Short case approach to cushing summary
 
Guides on Heart Disease
Guides on Heart DiseaseGuides on Heart Disease
Guides on Heart Disease
 
O&g examination
O&g examinationO&g examination
O&g examination
 
Guidelines of Epilepsy by NICE
Guidelines of Epilepsy by NICEGuidelines of Epilepsy by NICE
Guidelines of Epilepsy by NICE
 
Short case approach to parkinson's dz summary
Short case approach to parkinson's dz summaryShort case approach to parkinson's dz summary
Short case approach to parkinson's dz summary
 
Short case approach to acromegaly summary
Short case approach to acromegaly summaryShort case approach to acromegaly summary
Short case approach to acromegaly summary
 
Gina asthma management
Gina asthma managementGina asthma management
Gina asthma management
 
HIV/AIDS data Hub Asia Pacific -Malaysia 2014
HIV/AIDS data Hub Asia Pacific -Malaysia  2014HIV/AIDS data Hub Asia Pacific -Malaysia  2014
HIV/AIDS data Hub Asia Pacific -Malaysia 2014
 
Chronic renal failure concise long case approach & crf with fluid overload m...
Chronic renal failure concise long case approach  & crf with fluid overload m...Chronic renal failure concise long case approach  & crf with fluid overload m...
Chronic renal failure concise long case approach & crf with fluid overload m...
 
Neurological examination summary
Neurological examination summaryNeurological examination summary
Neurological examination summary
 

Similaire à Bohomolets 3rd year Surgery Peritonitis

Bohomolets 4th year Surgery Appendicitis
Bohomolets 4th year Surgery AppendicitisBohomolets 4th year Surgery Appendicitis
Bohomolets 4th year Surgery AppendicitisDr. Rubz
 
Peritonitis kawiz
Peritonitis kawizPeritonitis kawiz
Peritonitis kawizYoung Kawiz
 
Purulent diseases of serous cavities. Sepsis (lecture 13).pdf
Purulent diseases of serous cavities. Sepsis (lecture 13).pdfPurulent diseases of serous cavities. Sepsis (lecture 13).pdf
Purulent diseases of serous cavities. Sepsis (lecture 13).pdfshahajipawale0
 
Peritonitis and intra abdominal abscess
Peritonitis and intra abdominal abscessPeritonitis and intra abdominal abscess
Peritonitis and intra abdominal abscessSintayehu Asrat
 
Pyogenic liver abscess
Pyogenic liver abscessPyogenic liver abscess
Pyogenic liver abscessPratap Tiwari
 
Bohomolets 4th year Surgery Complication of Appendicitis
Bohomolets 4th year Surgery Complication of AppendicitisBohomolets 4th year Surgery Complication of Appendicitis
Bohomolets 4th year Surgery Complication of AppendicitisDr. Rubz
 
Bohomolets Oncology Lecture Methodical #2
Bohomolets Oncology Lecture Methodical #2Bohomolets Oncology Lecture Methodical #2
Bohomolets Oncology Lecture Methodical #2Dr. Rubz
 
GI10. Peritonitis 202bbbbbbbbbbbb1 (2).pptx
GI10. Peritonitis 202bbbbbbbbbbbb1 (2).pptxGI10. Peritonitis 202bbbbbbbbbbbb1 (2).pptx
GI10. Peritonitis 202bbbbbbbbbbbb1 (2).pptxmekuriatadesse
 
Systemic Surgery 2nd lecture The acute abdomen (1).pptx
Systemic Surgery 2nd lecture The acute abdomen (1).pptxSystemic Surgery 2nd lecture The acute abdomen (1).pptx
Systemic Surgery 2nd lecture The acute abdomen (1).pptxHassan25409
 
A RETROSPECTIVE ANALYSIS IN TERTIARY HOSPITAL FOR SURGICAL SITE INFECTIONS AF...
A RETROSPECTIVE ANALYSIS IN TERTIARY HOSPITAL FOR SURGICAL SITE INFECTIONS AF...A RETROSPECTIVE ANALYSIS IN TERTIARY HOSPITAL FOR SURGICAL SITE INFECTIONS AF...
A RETROSPECTIVE ANALYSIS IN TERTIARY HOSPITAL FOR SURGICAL SITE INFECTIONS AF...indexPub
 
Acs0413 Decortication And Pleurectomy
Acs0413 Decortication And PleurectomyAcs0413 Decortication And Pleurectomy
Acs0413 Decortication And Pleurectomymedbookonline
 
Bohomolets 4th year Surgery Exam Objective
Bohomolets 4th year Surgery Exam ObjectiveBohomolets 4th year Surgery Exam Objective
Bohomolets 4th year Surgery Exam ObjectiveDr. Rubz
 
Areas of the abdomen
Areas of the abdomenAreas of the abdomen
Areas of the abdomenEimaan Ktk
 
INTESTINAL FISTULA.pdf
INTESTINAL FISTULA.pdfINTESTINAL FISTULA.pdf
INTESTINAL FISTULA.pdfShapi. MD
 
Gossypiboma: A Diagnostic Challenge but a Surgeon's Nightmare.
Gossypiboma: A Diagnostic Challenge but a Surgeon's Nightmare.Gossypiboma: A Diagnostic Challenge but a Surgeon's Nightmare.
Gossypiboma: A Diagnostic Challenge but a Surgeon's Nightmare.KETAN VAGHOLKAR
 
Acs0536 Procedures For Rectal Prolapse 2004
Acs0536 Procedures For Rectal Prolapse 2004Acs0536 Procedures For Rectal Prolapse 2004
Acs0536 Procedures For Rectal Prolapse 2004medbookonline
 

Similaire à Bohomolets 3rd year Surgery Peritonitis (20)

Bohomolets 4th year Surgery Appendicitis
Bohomolets 4th year Surgery AppendicitisBohomolets 4th year Surgery Appendicitis
Bohomolets 4th year Surgery Appendicitis
 
Peritonitis kawiz
Peritonitis kawizPeritonitis kawiz
Peritonitis kawiz
 
Purulent diseases of serous cavities. Sepsis (lecture 13).pdf
Purulent diseases of serous cavities. Sepsis (lecture 13).pdfPurulent diseases of serous cavities. Sepsis (lecture 13).pdf
Purulent diseases of serous cavities. Sepsis (lecture 13).pdf
 
Peritonitis and intra abdominal abscess
Peritonitis and intra abdominal abscessPeritonitis and intra abdominal abscess
Peritonitis and intra abdominal abscess
 
Pyogenic liver abscess
Pyogenic liver abscessPyogenic liver abscess
Pyogenic liver abscess
 
12.Peritonitis.pdf
12.Peritonitis.pdf12.Peritonitis.pdf
12.Peritonitis.pdf
 
Peritonitis (lecture mogilevec e.v
Peritonitis (lecture mogilevec e.vPeritonitis (lecture mogilevec e.v
Peritonitis (lecture mogilevec e.v
 
Bohomolets 4th year Surgery Complication of Appendicitis
Bohomolets 4th year Surgery Complication of AppendicitisBohomolets 4th year Surgery Complication of Appendicitis
Bohomolets 4th year Surgery Complication of Appendicitis
 
Bohomolets Oncology Lecture Methodical #2
Bohomolets Oncology Lecture Methodical #2Bohomolets Oncology Lecture Methodical #2
Bohomolets Oncology Lecture Methodical #2
 
Pediatric surgery i
Pediatric surgery iPediatric surgery i
Pediatric surgery i
 
GI10. Peritonitis 202bbbbbbbbbbbb1 (2).pptx
GI10. Peritonitis 202bbbbbbbbbbbb1 (2).pptxGI10. Peritonitis 202bbbbbbbbbbbb1 (2).pptx
GI10. Peritonitis 202bbbbbbbbbbbb1 (2).pptx
 
Systemic Surgery 2nd lecture The acute abdomen (1).pptx
Systemic Surgery 2nd lecture The acute abdomen (1).pptxSystemic Surgery 2nd lecture The acute abdomen (1).pptx
Systemic Surgery 2nd lecture The acute abdomen (1).pptx
 
A RETROSPECTIVE ANALYSIS IN TERTIARY HOSPITAL FOR SURGICAL SITE INFECTIONS AF...
A RETROSPECTIVE ANALYSIS IN TERTIARY HOSPITAL FOR SURGICAL SITE INFECTIONS AF...A RETROSPECTIVE ANALYSIS IN TERTIARY HOSPITAL FOR SURGICAL SITE INFECTIONS AF...
A RETROSPECTIVE ANALYSIS IN TERTIARY HOSPITAL FOR SURGICAL SITE INFECTIONS AF...
 
Acs0413 Decortication And Pleurectomy
Acs0413 Decortication And PleurectomyAcs0413 Decortication And Pleurectomy
Acs0413 Decortication And Pleurectomy
 
Bohomolets 4th year Surgery Exam Objective
Bohomolets 4th year Surgery Exam ObjectiveBohomolets 4th year Surgery Exam Objective
Bohomolets 4th year Surgery Exam Objective
 
Peritonitis.pptx
Peritonitis.pptxPeritonitis.pptx
Peritonitis.pptx
 
Areas of the abdomen
Areas of the abdomenAreas of the abdomen
Areas of the abdomen
 
INTESTINAL FISTULA.pdf
INTESTINAL FISTULA.pdfINTESTINAL FISTULA.pdf
INTESTINAL FISTULA.pdf
 
Gossypiboma: A Diagnostic Challenge but a Surgeon's Nightmare.
Gossypiboma: A Diagnostic Challenge but a Surgeon's Nightmare.Gossypiboma: A Diagnostic Challenge but a Surgeon's Nightmare.
Gossypiboma: A Diagnostic Challenge but a Surgeon's Nightmare.
 
Acs0536 Procedures For Rectal Prolapse 2004
Acs0536 Procedures For Rectal Prolapse 2004Acs0536 Procedures For Rectal Prolapse 2004
Acs0536 Procedures For Rectal Prolapse 2004
 

Plus de Dr. Rubz

HIV discrimination among health providers in Malaysia by Dr Rubz
HIV discrimination among health providers in Malaysia by Dr RubzHIV discrimination among health providers in Malaysia by Dr Rubz
HIV discrimination among health providers in Malaysia by Dr RubzDr. Rubz
 
Regional Overview in HIV by Steve Kraus
Regional Overview in HIV by Steve KrausRegional Overview in HIV by Steve Kraus
Regional Overview in HIV by Steve KrausDr. Rubz
 
Game Changer by Dr Shaari Ngadiman
Game Changer by Dr Shaari NgadimanGame Changer by Dr Shaari Ngadiman
Game Changer by Dr Shaari NgadimanDr. Rubz
 
Pre and post HIV counseling (VCT)
Pre and post HIV counseling (VCT)Pre and post HIV counseling (VCT)
Pre and post HIV counseling (VCT)Dr. Rubz
 
Ulc auction final
Ulc auction finalUlc auction final
Ulc auction finalDr. Rubz
 
Testicular cancer for public awareness by Dr Rubz
Testicular cancer for public awareness by Dr RubzTesticular cancer for public awareness by Dr Rubz
Testicular cancer for public awareness by Dr RubzDr. Rubz
 
Prostate cancer for public awareness by DR RUBZ
Prostate cancer for public awareness by DR RUBZProstate cancer for public awareness by DR RUBZ
Prostate cancer for public awareness by DR RUBZDr. Rubz
 
Breast Cancer for public awareness by Dr Rubz
Breast Cancer for public awareness by Dr  RubzBreast Cancer for public awareness by Dr  Rubz
Breast Cancer for public awareness by Dr RubzDr. Rubz
 
Sex work presentation 9.18.13a
Sex work presentation 9.18.13aSex work presentation 9.18.13a
Sex work presentation 9.18.13aDr. Rubz
 
Rapid interpretation of ECG
Rapid interpretation of ECGRapid interpretation of ECG
Rapid interpretation of ECGDr. Rubz
 
Hernia by Dr. Rubzzz
Hernia by Dr. RubzzzHernia by Dr. Rubzzz
Hernia by Dr. RubzzzDr. Rubz
 
Benign breast disease by Dr. Kong
Benign breast disease by Dr. KongBenign breast disease by Dr. Kong
Benign breast disease by Dr. KongDr. Rubz
 
Breast CA by Dr. Celine Tey
Breast CA by Dr. Celine TeyBreast CA by Dr. Celine Tey
Breast CA by Dr. Celine TeyDr. Rubz
 
Other scrotal swelling by Dr. Teo
Other scrotal swelling by Dr. TeoOther scrotal swelling by Dr. Teo
Other scrotal swelling by Dr. TeoDr. Rubz
 
Ventral hernia by Dr Teo
Ventral hernia by Dr TeoVentral hernia by Dr Teo
Ventral hernia by Dr TeoDr. Rubz
 
Testicular torsion by Dr Teo
Testicular torsion by Dr TeoTesticular torsion by Dr Teo
Testicular torsion by Dr TeoDr. Rubz
 
Uk malaria treatment guideline
Uk malaria treatment guidelineUk malaria treatment guideline
Uk malaria treatment guidelineDr. Rubz
 
Tuberculosis summary
Tuberculosis summaryTuberculosis summary
Tuberculosis summaryDr. Rubz
 
Shock summary
Shock summaryShock summary
Shock summaryDr. Rubz
 
Stroke ( concise long case approach ) summary
Stroke ( concise long case approach ) summaryStroke ( concise long case approach ) summary
Stroke ( concise long case approach ) summaryDr. Rubz
 

Plus de Dr. Rubz (20)

HIV discrimination among health providers in Malaysia by Dr Rubz
HIV discrimination among health providers in Malaysia by Dr RubzHIV discrimination among health providers in Malaysia by Dr Rubz
HIV discrimination among health providers in Malaysia by Dr Rubz
 
Regional Overview in HIV by Steve Kraus
Regional Overview in HIV by Steve KrausRegional Overview in HIV by Steve Kraus
Regional Overview in HIV by Steve Kraus
 
Game Changer by Dr Shaari Ngadiman
Game Changer by Dr Shaari NgadimanGame Changer by Dr Shaari Ngadiman
Game Changer by Dr Shaari Ngadiman
 
Pre and post HIV counseling (VCT)
Pre and post HIV counseling (VCT)Pre and post HIV counseling (VCT)
Pre and post HIV counseling (VCT)
 
Ulc auction final
Ulc auction finalUlc auction final
Ulc auction final
 
Testicular cancer for public awareness by Dr Rubz
Testicular cancer for public awareness by Dr RubzTesticular cancer for public awareness by Dr Rubz
Testicular cancer for public awareness by Dr Rubz
 
Prostate cancer for public awareness by DR RUBZ
Prostate cancer for public awareness by DR RUBZProstate cancer for public awareness by DR RUBZ
Prostate cancer for public awareness by DR RUBZ
 
Breast Cancer for public awareness by Dr Rubz
Breast Cancer for public awareness by Dr  RubzBreast Cancer for public awareness by Dr  Rubz
Breast Cancer for public awareness by Dr Rubz
 
Sex work presentation 9.18.13a
Sex work presentation 9.18.13aSex work presentation 9.18.13a
Sex work presentation 9.18.13a
 
Rapid interpretation of ECG
Rapid interpretation of ECGRapid interpretation of ECG
Rapid interpretation of ECG
 
Hernia by Dr. Rubzzz
Hernia by Dr. RubzzzHernia by Dr. Rubzzz
Hernia by Dr. Rubzzz
 
Benign breast disease by Dr. Kong
Benign breast disease by Dr. KongBenign breast disease by Dr. Kong
Benign breast disease by Dr. Kong
 
Breast CA by Dr. Celine Tey
Breast CA by Dr. Celine TeyBreast CA by Dr. Celine Tey
Breast CA by Dr. Celine Tey
 
Other scrotal swelling by Dr. Teo
Other scrotal swelling by Dr. TeoOther scrotal swelling by Dr. Teo
Other scrotal swelling by Dr. Teo
 
Ventral hernia by Dr Teo
Ventral hernia by Dr TeoVentral hernia by Dr Teo
Ventral hernia by Dr Teo
 
Testicular torsion by Dr Teo
Testicular torsion by Dr TeoTesticular torsion by Dr Teo
Testicular torsion by Dr Teo
 
Uk malaria treatment guideline
Uk malaria treatment guidelineUk malaria treatment guideline
Uk malaria treatment guideline
 
Tuberculosis summary
Tuberculosis summaryTuberculosis summary
Tuberculosis summary
 
Shock summary
Shock summaryShock summary
Shock summary
 
Stroke ( concise long case approach ) summary
Stroke ( concise long case approach ) summaryStroke ( concise long case approach ) summary
Stroke ( concise long case approach ) summary
 

Dernier

Russian Call Girls Service Jaipur {8445551418} ❤️PALLAVI VIP Jaipur Call Gir...
Russian Call Girls Service  Jaipur {8445551418} ❤️PALLAVI VIP Jaipur Call Gir...Russian Call Girls Service  Jaipur {8445551418} ❤️PALLAVI VIP Jaipur Call Gir...
Russian Call Girls Service Jaipur {8445551418} ❤️PALLAVI VIP Jaipur Call Gir...parulsinha
 
Call Girls Gwalior Just Call 8617370543 Top Class Call Girl Service Available
Call Girls Gwalior Just Call 8617370543 Top Class Call Girl Service AvailableCall Girls Gwalior Just Call 8617370543 Top Class Call Girl Service Available
Call Girls Gwalior Just Call 8617370543 Top Class Call Girl Service AvailableDipal Arora
 
Best Rate (Patna ) Call Girls Patna ⟟ 8617370543 ⟟ High Class Call Girl In 5 ...
Best Rate (Patna ) Call Girls Patna ⟟ 8617370543 ⟟ High Class Call Girl In 5 ...Best Rate (Patna ) Call Girls Patna ⟟ 8617370543 ⟟ High Class Call Girl In 5 ...
Best Rate (Patna ) Call Girls Patna ⟟ 8617370543 ⟟ High Class Call Girl In 5 ...Dipal Arora
 
Top Rated Hyderabad Call Girls Chintal ⟟ 9332606886 ⟟ Call Me For Genuine Se...
Top Rated  Hyderabad Call Girls Chintal ⟟ 9332606886 ⟟ Call Me For Genuine Se...Top Rated  Hyderabad Call Girls Chintal ⟟ 9332606886 ⟟ Call Me For Genuine Se...
Top Rated Hyderabad Call Girls Chintal ⟟ 9332606886 ⟟ Call Me For Genuine Se...chandars293
 
Call Girls Kakinada Just Call 9907093804 Top Class Call Girl Service Available
Call Girls Kakinada Just Call 9907093804 Top Class Call Girl Service AvailableCall Girls Kakinada Just Call 9907093804 Top Class Call Girl Service Available
Call Girls Kakinada Just Call 9907093804 Top Class Call Girl Service AvailableDipal Arora
 
VIP Service Call Girls Sindhi Colony 📳 7877925207 For 18+ VIP Call Girl At Th...
VIP Service Call Girls Sindhi Colony 📳 7877925207 For 18+ VIP Call Girl At Th...VIP Service Call Girls Sindhi Colony 📳 7877925207 For 18+ VIP Call Girl At Th...
VIP Service Call Girls Sindhi Colony 📳 7877925207 For 18+ VIP Call Girl At Th...jageshsingh5554
 
Mumbai ] (Call Girls) in Mumbai 10k @ I'm VIP Independent Escorts Girls 98333...
Mumbai ] (Call Girls) in Mumbai 10k @ I'm VIP Independent Escorts Girls 98333...Mumbai ] (Call Girls) in Mumbai 10k @ I'm VIP Independent Escorts Girls 98333...
Mumbai ] (Call Girls) in Mumbai 10k @ I'm VIP Independent Escorts Girls 98333...Ishani Gupta
 
Manyata Tech Park ( Call Girls ) Bangalore ✔ 6297143586 ✔ Hot Model With Sexy...
Manyata Tech Park ( Call Girls ) Bangalore ✔ 6297143586 ✔ Hot Model With Sexy...Manyata Tech Park ( Call Girls ) Bangalore ✔ 6297143586 ✔ Hot Model With Sexy...
Manyata Tech Park ( Call Girls ) Bangalore ✔ 6297143586 ✔ Hot Model With Sexy...vidya singh
 
Top Rated Hyderabad Call Girls Erragadda ⟟ 9332606886 ⟟ Call Me For Genuine ...
Top Rated  Hyderabad Call Girls Erragadda ⟟ 9332606886 ⟟ Call Me For Genuine ...Top Rated  Hyderabad Call Girls Erragadda ⟟ 9332606886 ⟟ Call Me For Genuine ...
Top Rated Hyderabad Call Girls Erragadda ⟟ 9332606886 ⟟ Call Me For Genuine ...chandars293
 
💕SONAM KUMAR💕Premium Call Girls Jaipur ↘️9257276172 ↙️One Night Stand With Lo...
💕SONAM KUMAR💕Premium Call Girls Jaipur ↘️9257276172 ↙️One Night Stand With Lo...💕SONAM KUMAR💕Premium Call Girls Jaipur ↘️9257276172 ↙️One Night Stand With Lo...
💕SONAM KUMAR💕Premium Call Girls Jaipur ↘️9257276172 ↙️One Night Stand With Lo...khalifaescort01
 
Call Girls Visakhapatnam Just Call 8250077686 Top Class Call Girl Service Ava...
Call Girls Visakhapatnam Just Call 8250077686 Top Class Call Girl Service Ava...Call Girls Visakhapatnam Just Call 8250077686 Top Class Call Girl Service Ava...
Call Girls Visakhapatnam Just Call 8250077686 Top Class Call Girl Service Ava...Dipal Arora
 
Most Beautiful Call Girl in Bangalore Contact on Whatsapp
Most Beautiful Call Girl in Bangalore Contact on WhatsappMost Beautiful Call Girl in Bangalore Contact on Whatsapp
Most Beautiful Call Girl in Bangalore Contact on WhatsappInaaya Sharma
 
Top Rated Bangalore Call Girls Majestic ⟟ 9332606886 ⟟ Call Me For Genuine S...
Top Rated Bangalore Call Girls Majestic ⟟  9332606886 ⟟ Call Me For Genuine S...Top Rated Bangalore Call Girls Majestic ⟟  9332606886 ⟟ Call Me For Genuine S...
Top Rated Bangalore Call Girls Majestic ⟟ 9332606886 ⟟ Call Me For Genuine S...narwatsonia7
 
Call Girls Rishikesh Just Call 8250077686 Top Class Call Girl Service Available
Call Girls Rishikesh Just Call 8250077686 Top Class Call Girl Service AvailableCall Girls Rishikesh Just Call 8250077686 Top Class Call Girl Service Available
Call Girls Rishikesh Just Call 8250077686 Top Class Call Girl Service AvailableDipal Arora
 
Call Girls Vasai Virar Just Call 9630942363 Top Class Call Girl Service Avail...
Call Girls Vasai Virar Just Call 9630942363 Top Class Call Girl Service Avail...Call Girls Vasai Virar Just Call 9630942363 Top Class Call Girl Service Avail...
Call Girls Vasai Virar Just Call 9630942363 Top Class Call Girl Service Avail...GENUINE ESCORT AGENCY
 
Top Quality Call Girl Service Kalyanpur 6378878445 Available Call Girls Any Time
Top Quality Call Girl Service Kalyanpur 6378878445 Available Call Girls Any TimeTop Quality Call Girl Service Kalyanpur 6378878445 Available Call Girls Any Time
Top Quality Call Girl Service Kalyanpur 6378878445 Available Call Girls Any TimeCall Girls Delhi
 
Top Rated Bangalore Call Girls Mg Road ⟟ 9332606886 ⟟ Call Me For Genuine S...
Top Rated Bangalore Call Girls Mg Road ⟟   9332606886 ⟟ Call Me For Genuine S...Top Rated Bangalore Call Girls Mg Road ⟟   9332606886 ⟟ Call Me For Genuine S...
Top Rated Bangalore Call Girls Mg Road ⟟ 9332606886 ⟟ Call Me For Genuine S...narwatsonia7
 
The Most Attractive Hyderabad Call Girls Kothapet 𖠋 9332606886 𖠋 Will You Mis...
The Most Attractive Hyderabad Call Girls Kothapet 𖠋 9332606886 𖠋 Will You Mis...The Most Attractive Hyderabad Call Girls Kothapet 𖠋 9332606886 𖠋 Will You Mis...
The Most Attractive Hyderabad Call Girls Kothapet 𖠋 9332606886 𖠋 Will You Mis...chandars293
 
Night 7k to 12k Navi Mumbai Call Girl Photo 👉 BOOK NOW 9833363713 👈 ♀️ night ...
Night 7k to 12k Navi Mumbai Call Girl Photo 👉 BOOK NOW 9833363713 👈 ♀️ night ...Night 7k to 12k Navi Mumbai Call Girl Photo 👉 BOOK NOW 9833363713 👈 ♀️ night ...
Night 7k to 12k Navi Mumbai Call Girl Photo 👉 BOOK NOW 9833363713 👈 ♀️ night ...aartirawatdelhi
 
Pondicherry Call Girls Book Now 9630942363 Top Class Pondicherry Escort Servi...
Pondicherry Call Girls Book Now 9630942363 Top Class Pondicherry Escort Servi...Pondicherry Call Girls Book Now 9630942363 Top Class Pondicherry Escort Servi...
Pondicherry Call Girls Book Now 9630942363 Top Class Pondicherry Escort Servi...GENUINE ESCORT AGENCY
 

Dernier (20)

Russian Call Girls Service Jaipur {8445551418} ❤️PALLAVI VIP Jaipur Call Gir...
Russian Call Girls Service  Jaipur {8445551418} ❤️PALLAVI VIP Jaipur Call Gir...Russian Call Girls Service  Jaipur {8445551418} ❤️PALLAVI VIP Jaipur Call Gir...
Russian Call Girls Service Jaipur {8445551418} ❤️PALLAVI VIP Jaipur Call Gir...
 
Call Girls Gwalior Just Call 8617370543 Top Class Call Girl Service Available
Call Girls Gwalior Just Call 8617370543 Top Class Call Girl Service AvailableCall Girls Gwalior Just Call 8617370543 Top Class Call Girl Service Available
Call Girls Gwalior Just Call 8617370543 Top Class Call Girl Service Available
 
Best Rate (Patna ) Call Girls Patna ⟟ 8617370543 ⟟ High Class Call Girl In 5 ...
Best Rate (Patna ) Call Girls Patna ⟟ 8617370543 ⟟ High Class Call Girl In 5 ...Best Rate (Patna ) Call Girls Patna ⟟ 8617370543 ⟟ High Class Call Girl In 5 ...
Best Rate (Patna ) Call Girls Patna ⟟ 8617370543 ⟟ High Class Call Girl In 5 ...
 
Top Rated Hyderabad Call Girls Chintal ⟟ 9332606886 ⟟ Call Me For Genuine Se...
Top Rated  Hyderabad Call Girls Chintal ⟟ 9332606886 ⟟ Call Me For Genuine Se...Top Rated  Hyderabad Call Girls Chintal ⟟ 9332606886 ⟟ Call Me For Genuine Se...
Top Rated Hyderabad Call Girls Chintal ⟟ 9332606886 ⟟ Call Me For Genuine Se...
 
Call Girls Kakinada Just Call 9907093804 Top Class Call Girl Service Available
Call Girls Kakinada Just Call 9907093804 Top Class Call Girl Service AvailableCall Girls Kakinada Just Call 9907093804 Top Class Call Girl Service Available
Call Girls Kakinada Just Call 9907093804 Top Class Call Girl Service Available
 
VIP Service Call Girls Sindhi Colony 📳 7877925207 For 18+ VIP Call Girl At Th...
VIP Service Call Girls Sindhi Colony 📳 7877925207 For 18+ VIP Call Girl At Th...VIP Service Call Girls Sindhi Colony 📳 7877925207 For 18+ VIP Call Girl At Th...
VIP Service Call Girls Sindhi Colony 📳 7877925207 For 18+ VIP Call Girl At Th...
 
Mumbai ] (Call Girls) in Mumbai 10k @ I'm VIP Independent Escorts Girls 98333...
Mumbai ] (Call Girls) in Mumbai 10k @ I'm VIP Independent Escorts Girls 98333...Mumbai ] (Call Girls) in Mumbai 10k @ I'm VIP Independent Escorts Girls 98333...
Mumbai ] (Call Girls) in Mumbai 10k @ I'm VIP Independent Escorts Girls 98333...
 
Manyata Tech Park ( Call Girls ) Bangalore ✔ 6297143586 ✔ Hot Model With Sexy...
Manyata Tech Park ( Call Girls ) Bangalore ✔ 6297143586 ✔ Hot Model With Sexy...Manyata Tech Park ( Call Girls ) Bangalore ✔ 6297143586 ✔ Hot Model With Sexy...
Manyata Tech Park ( Call Girls ) Bangalore ✔ 6297143586 ✔ Hot Model With Sexy...
 
Top Rated Hyderabad Call Girls Erragadda ⟟ 9332606886 ⟟ Call Me For Genuine ...
Top Rated  Hyderabad Call Girls Erragadda ⟟ 9332606886 ⟟ Call Me For Genuine ...Top Rated  Hyderabad Call Girls Erragadda ⟟ 9332606886 ⟟ Call Me For Genuine ...
Top Rated Hyderabad Call Girls Erragadda ⟟ 9332606886 ⟟ Call Me For Genuine ...
 
💕SONAM KUMAR💕Premium Call Girls Jaipur ↘️9257276172 ↙️One Night Stand With Lo...
💕SONAM KUMAR💕Premium Call Girls Jaipur ↘️9257276172 ↙️One Night Stand With Lo...💕SONAM KUMAR💕Premium Call Girls Jaipur ↘️9257276172 ↙️One Night Stand With Lo...
💕SONAM KUMAR💕Premium Call Girls Jaipur ↘️9257276172 ↙️One Night Stand With Lo...
 
Call Girls Visakhapatnam Just Call 8250077686 Top Class Call Girl Service Ava...
Call Girls Visakhapatnam Just Call 8250077686 Top Class Call Girl Service Ava...Call Girls Visakhapatnam Just Call 8250077686 Top Class Call Girl Service Ava...
Call Girls Visakhapatnam Just Call 8250077686 Top Class Call Girl Service Ava...
 
Most Beautiful Call Girl in Bangalore Contact on Whatsapp
Most Beautiful Call Girl in Bangalore Contact on WhatsappMost Beautiful Call Girl in Bangalore Contact on Whatsapp
Most Beautiful Call Girl in Bangalore Contact on Whatsapp
 
Top Rated Bangalore Call Girls Majestic ⟟ 9332606886 ⟟ Call Me For Genuine S...
Top Rated Bangalore Call Girls Majestic ⟟  9332606886 ⟟ Call Me For Genuine S...Top Rated Bangalore Call Girls Majestic ⟟  9332606886 ⟟ Call Me For Genuine S...
Top Rated Bangalore Call Girls Majestic ⟟ 9332606886 ⟟ Call Me For Genuine S...
 
Call Girls Rishikesh Just Call 8250077686 Top Class Call Girl Service Available
Call Girls Rishikesh Just Call 8250077686 Top Class Call Girl Service AvailableCall Girls Rishikesh Just Call 8250077686 Top Class Call Girl Service Available
Call Girls Rishikesh Just Call 8250077686 Top Class Call Girl Service Available
 
Call Girls Vasai Virar Just Call 9630942363 Top Class Call Girl Service Avail...
Call Girls Vasai Virar Just Call 9630942363 Top Class Call Girl Service Avail...Call Girls Vasai Virar Just Call 9630942363 Top Class Call Girl Service Avail...
Call Girls Vasai Virar Just Call 9630942363 Top Class Call Girl Service Avail...
 
Top Quality Call Girl Service Kalyanpur 6378878445 Available Call Girls Any Time
Top Quality Call Girl Service Kalyanpur 6378878445 Available Call Girls Any TimeTop Quality Call Girl Service Kalyanpur 6378878445 Available Call Girls Any Time
Top Quality Call Girl Service Kalyanpur 6378878445 Available Call Girls Any Time
 
Top Rated Bangalore Call Girls Mg Road ⟟ 9332606886 ⟟ Call Me For Genuine S...
Top Rated Bangalore Call Girls Mg Road ⟟   9332606886 ⟟ Call Me For Genuine S...Top Rated Bangalore Call Girls Mg Road ⟟   9332606886 ⟟ Call Me For Genuine S...
Top Rated Bangalore Call Girls Mg Road ⟟ 9332606886 ⟟ Call Me For Genuine S...
 
The Most Attractive Hyderabad Call Girls Kothapet 𖠋 9332606886 𖠋 Will You Mis...
The Most Attractive Hyderabad Call Girls Kothapet 𖠋 9332606886 𖠋 Will You Mis...The Most Attractive Hyderabad Call Girls Kothapet 𖠋 9332606886 𖠋 Will You Mis...
The Most Attractive Hyderabad Call Girls Kothapet 𖠋 9332606886 𖠋 Will You Mis...
 
Night 7k to 12k Navi Mumbai Call Girl Photo 👉 BOOK NOW 9833363713 👈 ♀️ night ...
Night 7k to 12k Navi Mumbai Call Girl Photo 👉 BOOK NOW 9833363713 👈 ♀️ night ...Night 7k to 12k Navi Mumbai Call Girl Photo 👉 BOOK NOW 9833363713 👈 ♀️ night ...
Night 7k to 12k Navi Mumbai Call Girl Photo 👉 BOOK NOW 9833363713 👈 ♀️ night ...
 
Pondicherry Call Girls Book Now 9630942363 Top Class Pondicherry Escort Servi...
Pondicherry Call Girls Book Now 9630942363 Top Class Pondicherry Escort Servi...Pondicherry Call Girls Book Now 9630942363 Top Class Pondicherry Escort Servi...
Pondicherry Call Girls Book Now 9630942363 Top Class Pondicherry Escort Servi...
 

Bohomolets 3rd year Surgery Peritonitis

  • 1. O.O. Bohomolets National Medical University CHAIR OF GENERAL SURGERY No. 1 Approved by methodical meeting of the chair "__"___2007, minutes No.___ Prof., MD, PhD. O.I. Dronov METHODICAL GUIDES FOR PRACTICAL LESSONS Materials for teachers and students Subject: Peritonitis: Inflammation of the peritoneal cavity. Classifications, clinics, diagnosis, prophylaxis Course 3 Faculties 2 and 3, faculty for educating the doctors for Ukrainian Army, faculty of medical psychology Duration: 4 academic hours Prepared by assistant M.D. S.V. Zemskov Kyiv 2007
  • 2. 2 I. Priorities of the subject: The concept of the relative and absolute indications for a surgery exists in the surgical practice. The absolute indication is the patient's condition which could not be corrected by the conservative means. Peritonitis or he threat of peritonitis is the most prevalent absolute indication for the urgent abdominal surgery. Therefore, the knowledge of the clinical features of peritonitis and the major principles of its treatment is of vital importance for the clinicians. II. The aims of training: А-1. Scope of the knowledge: - Anatomy and histology of the peritoneum and the greater omentum; - Resorptive and exudative properties of the peritoneum; - Biological and chemical characteristics of the secretions of the glands comprising the digestive system and other biological fluids of the body capable of contacting the peritoneum; - Features of the local and general inflammatory response of the peritoneum; - Clinical symptoms and syndromes peculiar to peritonitis А-2. Student should be acquainted with the following subjects: - Mechanisms of the development of localized and general peritonitis; - Extent of clinical manifestations depending on the etiology of peritonitis; - Pathogenesis of complications due to peritonitis; - Hazards of peritonitis and importance of the timely adequate treatment. А-3. Scope for practical skills: - Competent anamnesis for the assessment of the causes of peritonitis; - Use of physical examination techniques and assessment of pathognomonic symptoms of peritonitis; - Timely recognition of peritonitis and use of the require complex of the diagnostic procedures for confirmation / refusal of peritonitis diagnosis. А-4. List of practical technique to be trained: - Detection of Schotkin-Blumberg symptom; - Interpretation of the survey X-ray film of peritoneal cavity - Interpretation of the ultrasonographic data. III. Training and educational objectives: 1. To demonstrate the importance of the subject for the timely and adequate treatment of peritonitis. 2. To demand the adherence to the principles of the medical ethics and deontology. 3. To demonstrate the importance of the knowledge and skills in the field employing the practical examples.
  • 3. 3 IV. Interdisciplinary integration: Subject for study and Scope of knowledge Scope of skills corresponding chair Pathological states resulting in Detection and interpretation of General surgery with care complications manifesting as pathognomonic symptoms of for patients peritonitis peritonitis Interpretation of peritonitis Anatomical structure and probability in case of injury of Human anatomy relation of the peritoneal the specified organ located in organs to the peritoneum peritoneal cavity Structure and embryogenesis Histology and embryology of peritoneum Differential prescription of Microflora of the specific Microbiology antibacterial agents according to divisions of the intestines the etiology of peritonitis Exudative and resorptive Indications for drainage / lavage Physiology functions of peritoneum of the peritoneal cavity V. Content of training INTRODUCTION The peritoneum is a two-layered serous membrane that lines the abdominal cavity covering the inside walls of the abdomen (parietal peritoneum) and the abdominal organs (visceral peritoneum). Passing from the abdominal walls to the intra-abdominal organs and from one organ to another one, the peritoneum generated the folds and the mesenteries bordering the spaces, the sinuses, and the recesses. The peritoneal cavity is limited by the diaphragm from above, the pelvic diaphragm and the ilia from below, then backbone and the muscles of the back posteriorly, the rectus muscles of abdomen anteriorly, and the internal oblique and transverse muscles of abdomen anteriorly and laterally. The peritoneum is a semipermeable actively functioning membrane possessing several functions such as exudative-and-resorptive function and the barrier function (due to migrating and resident macrophages, immunoglobulins, and non-specific factors. Histologically, the peritoneum comprises six layers: mesothelium, basal membrane and four layers of elastic and collagenous fibers. On average, the thickness of the serous membrane is about 0.2 mm. The total area of the peritoneal cover amounts to 17000-20400 square centimeters, which practically equals to the area of the skin surface.
  • 4. 4 The exudative areas of the peritoneum are represented for the most part by the serous cover of the intestines. The most intensive exudation is in the duodenal area with the gradual decrease in caudal direction. The most intensive resorption is inherent to the peritoneum of the diaphragm, the greater omentum, the ileum, and the cecum. The amount of the fluid absorbed by peritoneum within one hour makes up to 8% of body mass. The greater omentum represents the duplicature of the peritoneum containing the fat with well-developed blood and lymphatic vessels providing for the protective function circumscribing the foci of the inflammation by means of the fibrin. The peritoneum is supplied by the blood from the basins of the vessels feeding the corresponding organs. The outflow of the venous blood is provided mainly through the portal system and partly through the caval system. The most intensive outflow of the lymph occurs from the surface of the greater omentum and the diaphragm. The visceral peritoneum has the vegetative innervation (parasympathetic and sympathetic) and practically is devoid of the somatic innervation. This is the reason why the visceral pains due to the irritation of the visceral peritoneum are not localized. The most sensitive are so called reflex areas of the root of mesentery, the areas of the celiac trunk, the pancreas, the ileocecal angle, and Douglas pouch. The parietal peritoneum is devoid of the somatic innervation. This fact explains the absence of the protective tension of the muscles of the anterior abdominal wall in the case of the inflammations in the small pelvis. PERITONITIS CLASSIFICATION (after Yu.M. Lopukhin and V.S. Saveliev) І. According to the clinical course: acute and chronic. ІІ. According to the route of penetration of bacteria into the peritoneal cavity: А. Primary peritonitis when the infection spreads by hematogenic or lymphogenic route or via the fallopian tubes. B. Secondary peritonitis when the infection penetrates due to the acute surgical conditions or the injury of the peritoneal organs. 1. Infectious and inflammatory peritonitis results from the following diseases of the peritoneal organs: - acute appendicitis, - acute cholecystitis, - acute ileus,
  • 5. 5 - acute pancreatitis, - thromboembolia of the mesenterial blood vessels, - diverticulitis, - intestinal tumors, - gynecological conditions. 2. Perforative peritonitis results from the perforations due to: - duodenal and gastric ulcers; - intestinal ulcerations associated with typhus, dysentery, tuberculosis, cancer, stress, etc.) - decubitus ulcer in the obturation ileus; - strangulation groove in strangulated intestinal obstruction in the foreign bodies of gastrointestinal tract; - intestinal necrosis in the strangulated hernia or thromboembolia of the mesenterial blood vessels. 3. Traumatic peritonitis develops in the open and closed abdominal traumas both with accompanying damage of the abdominal organs and without such damage. 4. Postoperative peritonitis results from: - the failure of the sutures of anastomosis after the abdominal surgery; - the infection of the peritoneal cavity in the course of the surgery; - the defects of the ligatures applied to the large areas of the omentum and the mesentery followed by the necrosis of the tissues distal to the ligature - the mechanical damage of the peritoneum: drying or the hemorrhage into the free abdominal cavity without the reliable hemostasis. ІІІ. According to the microbiological features 1. Bacterial peritonitis: non-specific caused by microflora of the gastrointestinal tract and specific caused by microflora, which is not associated with the gastrointestinal tract such as gonococci (Neіsserіa gonorrhoeae), pneumococci (Streptococcus pneumonіaе), hemolytic streptococci (Streptococcus pyogenes, Streptococcus vіrіdans), micobacteria of tuberculosis (Mycobacterіum tuberculosіs). 2. Aseptical peritonitis develops because of the exposure of peritoneum to the toxic and enzymatic agents of non-infectious origin (the blood, the bile, the gastric juice, the chylous fluid, the pancreatic juice, the urine, the aseptic necrosis if the internal organs). 3. Specific forms of peritonitis: Carcinomatous. Parasitic.
  • 6. 6 Rheumatoid. Granulomatous. ІV. According to the character of the peritoneal exudate. " Serous " Fibrinous " Purulent " Hemorrhagic V. According to the character of the lesions of the peritoneal surface 1. Depending on the circumscription, the following forms of peritonitis may be delineated: The circumscribed peritonitis may be represented as an abscess or infiltrate (for example, paravesical abscess or pericholecystitis as the complication of the gangrenous cholecystitis) Non-circumscribed peritonitis lacks the strict boundaries and the tendencies towards the circumscription. 2. Depending on the extent of the spread, the following forms of peritonitis may be delineated : Localized: confined only to one anatomical division of the abdominal cavity. Diffuse: extended to 2-5 anatomical divisions of the abdominal cavity. Generalized: total involvement of the peritoneum (6 and more divisions of the abdominal cavity. VІ. According to the phases of the development. 1. Reactive phase (first 24 h, 12 h for perforative peritonitis) 2. Toxic (24-72 h, 12-24 h for perforative peritonitis) 3. Terminal (more than 72 h, more than 24 h for perforative peritonitis). Definition of peritonitis as the surgical pathology Peritonitis is defined as the acute inflammation of the peritoneum. Peritonitis represents one of the most severe complication accompanying the diseases and the injuries of the intra-abdominal organs and may be delineated as the separate nosology with the characteristic clinical pattern and the complex of the severe pathophysiological reactions with the impairment of the whole homeostasis systems. Referring to the peritonitis and its surgical treatment, one bears in mind acute, secondary, non-specific peritonitis being the cause of complications of about 15-20 % of all the acute pathologies of the abdominal organs. Among the forms of the acute peritonitis, the acute appendicitis with the incidence of 30-65% ranks the first followed by the perforated ulcer (7-15 %), the acute cholecystitis (10-12 %), the gynecological pathologies (3-12 %), the ileus (3-5 %), the pancreatitis (1 %), and postoperative peritonitis (1 %).
  • 7. 7 The chronic peritonitis may develop in tuberculosis, mycosis, carcinomatosis, also in ascitic conditions and in the presence of the numerous syphilitic gummas. The aseptic peritonitis develops - Upon irritation of the peritoneum with various chemical agents (iodine, ethanol, non- isotonic solutions, some antiseptics, enzymes); - Upon irritation of the peritoneum with the aggressive biological fluids (urine, bile, pancreatic or gastric juice, the content of the hydatid cyst), - Upon the extensive ligation of the tissues. All the forms indicated above make up to less than 1 % of all peritonitis cases, the rest being attributed to the acute secondary peritonitis. CLASSIFICATION OF SURGICAL PERITONITIS The general classification is rather bulky for everyday use. That is why in so called working classifications the cause of the peritonitis is given (the perforated ulcer or acute gangrenous perforated appendicitis) with accompanying morphological description (diffuse, fibrinous-purulent peritonitis). The words such as "secondary" and "infectious non-specific" are usually being avoided). In final diagnosis, the complications resulting from the infectious (septic) processes and postoperative complications are referred to (if any). Circumscription and extent of spreading: The redundant terms (circumscribed – local or non-circumscribed – diffuse) should not be used together. The area of the lesion should be strictly indicated. Phases of the development: We use the pathogenetic classification after K. Simonian: Reactive phase: The manifestations of the peritonitis are caused by the activated defense systems such as the massive release of the kinins in the response to the primary infectious aggression. Therefore, when the source of infection is eliminated, in reactive phase the radical surgery is possible with anastomoses being applied to the abdominal organs. In the inflammatory process, the reactive phase lasts up to 24 hours (in perforative peritonitis up to 12 hours). Upon the source of the peritonitis is eliminated (the perforated ulcer, for example) in the early (reactive) phase, the peritonitis does not progress as usual and the lethality is rather low. Toxic phase is characterized by the alterations in the functions of all systems of the body because of the syndrome of the systemic inflammatory response. The toxic phase is evident in 12-24 hours after the onset of the disease in perforative processes and 24-48 hours in the inflammatory processes. The lethality is 20% and higher.
  • 8. 8 Terminal phase is characterized by the development of the polyorgan insufficiency syndrome representing the depletion of the functional reserves of the major systems of the body. The toxic phase is evident in 24-36 hours in the perforative peritonitis and in 48-72 hours in the inflammatory processes. The lethality in the patients with polyorgan insufficiency syndrome is about 90%. Complications The local and general complications may be delineated. The local complications comprise pylephlebitis, the multiple abscesses of the liver. The general complications comprise the liver insufficiency, the toxic encephalopathy, the renal failure, the cardiovascular failure. The predominant part of the peritoneum covers the intestines. It is naturally, therefore, that the toxic products flowing out with blood or lymph impair microcirculation in the intestinal wall and neuromuscular transmission resulting in decreasing and arresting the peristalsis (intestinal paresis). The intestinal content is a good milieu for the development of bacteria including the anaerobic ones since the aerobic bacteria realize the available oxygen. The arrest of the intestinal passage results in the increasing microbial content in the middle and upper thirds of the small intestine where the bacteria are normally absent. The serous edema extends to the whole intestinal wall. The mucosa loses the barrier function. Ads a result, the bacteria and their metabolites are absorbed into the lymph and the blood circulation not only from the peritoneal cavity but from the intestine as well. Therefore, since the onset of the intestinal paresis in peritonitis, the intoxication and dehydration of the body progress rapidly. The increase in the toxic load to the liver results in then rapid depletion of its detoxifying function and other metabolic functions as well. The penetration of the bacteria and toxins into the caval system results in extra loading to the lungs resulting in the development of the inflammation in the lung tissue. As a consequence, the respiratory function is impaired with the development of the respiratory distress syndrome resulting finally in the pneumonia. Microbiological features of peritonitis As a rule, the exudate is infected with variety of bacterial species inhabiting the cavities and the surfaces of the human body. The most prevalent batteries are the Escherichia coli, Enterococcus, and Proteus sp. The putrefactive odor and the sordid of the exudate suggest that the predominant microflora relates to the facultative anaerobic bacteria (so-called non-clostridial anaerobic microflora).
  • 9. 9 Earlier, gas chromatography was used as the express method for detecting the bacteria comprising the exudates based on the analysis of the specific bacterial metabolites – volatile fatty acids (propionic, butyric, valeric). The up-to-date techniques of detection and identification are based on the immunocytochemistry and polymerase chain reaction. Upon the prolonged treatment, the nosocomial infection caused by penicillinase- producing cocci or Pseudomonas aerugіnosa may be evident. The results of the commonly used bacteriologic techniques are evident in three days. Therefore, the initial antibiotic therapy should be based on the previous experience in the field. The following findings should be taken into account: – Gram-positive cocci are the major bacterial species in the case of the localization of the primary focus in the upper division of the gastrointestinal tract (the stomach, the duodenum, the bile ducts, the pancreas, upper third of the small intestine). The probability of the involvement of the anaerobic bacteria is rather low (10-15%) and depends entirely on the duration of the process. – Gram-negative bacilli and non-clostridial anaerobic bacteria prevail in the perforation of the small intestine and the appendix as well as in the cases of peritonitis with the accompanying ileus. – Escherichia coli ranks among the first in the development of the peritonitis when the perforation is not available. SYMPTOMATICS AND DIAGNOSIS OF PERITONITIS The diagnosis of the peritonitis is based on the complex assessment of the anamnesis, the local symptoms, the presence of the inflammation and intoxication, and the specific complications. Complaints: The steady blunt pain throughout the abdomen increasing upon the movements and the breathing. The irradiation into the shoulder is possible upon the irritation of the diaphragm. The general sickness is evident with frequent nausea and stool retention. Anamnesis The time passed since the onset of the symptoms should be clarified. The dynamics of the character and the localization of the pain, the dynamics of the toxic manifestations are followed. In most cases, the signs of the previous disease (appendicitis, cholecystitis, acute condition of the ulcer) resulting in peritonitis are evident. Frequently, in the setting of some improvement of the general state and decreasing of the abdominal pains, the sudden increase of the pain occurs
  • 10. 10 which becomes more extensive. Since this moment, the general state of the patient deteriorates, the dryness in mouth and the thirst appear, the palpitation increases. General examination As usual, the patient lies in supine position with the knees bent. The attempts to change the pose or to rise fail due to the increasing pain. When the patient sits, the attempt to lie down results in increasing abdominal pain and the pain irradiating to the shoulders due to the irritation of the diaphragmatic nerve. The patient is forced to take up the previous sitting position (the tumbler toy symptom). The patient speaks quietly and does not shout claiming the attention. Such a behavior of the patient especially the breathing accompanied by the quiet moaning is a sign of particular concern for the doctor. All the components of the infectious process and the intoxication such as the temperature, the tachycardia, the rate, and the deepness of breathing, the changes in blood pressure, the dryness of the tongue and the internal buccal surface should be taken into account. The loss of consciousness should be of special concern signifying the severe intoxication. The tachycardia with heart rate of 100-120 beats per minutes is evident with normal or decreased blood pressure and the increased respiration rate (20-24 breathings per minute). The toxic encephalopathy may be manifested both as the inhibition and the excitation in the form of delirium. The paleness of the skin and cutis marmorata reflects the deep disorder of microcirculation. Specific examination The abdomen is usually symmetrical, somehow distended, is not involved in breathing movements. By palpation, the diffuse pain, the tension of the abdomen, and Mendel and Schotkin-Blumberg symptoms are revealed. The peristalsis sounds are usually weakened or absent. The presence of the free gas should be checked by the disappearance of the liver dullness. The presence of the free liquid is proved by the dullness of the percussion sound in the lateral divisions disappearing upon turning the patient to the side. In rectal and vaginal examination, the overhang of the fornices and the tenderness due to the accumulation of the inflammatory exudates are frequently evident. In general blood analysis, the pronounced leukocytosis and the shift towards the immature forms are present.
  • 11. 11 Biochemically, normo- or hyperglycemia, the moderate increase of urea, creatinine, the alanine aminotransferase, aspartate aminotransferase, alkaline phosphatase activities are detected; the syndrome of the disseminated intravascular blood coagulation may develop. Laboratory examination The moderate leukocytosis (14-20 х 109/L) with the shift of the formula to the left, lympho- and monopenia, aeosinophilia, thrombocytopenia are evident. For the objective assessment of the intoxication extent, Calf-Calif leukocytic index of intoxication is calciulated with its value of 4 characteristic of the reactive phase, the value of 8 characteristic of the toxic phase, and the value of 12-18 characteristic of the terminal phase. The increase of the hematocrit, the increased content of urea and creatinin, the increased activity of transaminases, the elevation of bilirubin, glucose, lactate, the alteration of the parameters of coagulation system, the changes of acid-alkaline state, the increase in the blood partial pressure of CO2 and the decrease in the blood partial pressure of O2 are the signs of the impaired functions of various organs and systems. Additional examinations The presence of the free fluid in the abdominal cavity may be confirmed sonographically. X-ray examination may be useful for detecting free gases, the signs of ileus (Kloiber bowls), the high position of the diaphragmatic cupolas and the limitation of their mobility due to the tension of the abdominal muscles (the muscular defense). In the severe cases, the respiratory distress syndrome develops. In doubtful cases, laparoscopy may be performed for clarifying the source and the extent of the source of the peritonitis. The diagnosis of peritonitis may be confirmed by laparocentesis by the presence of the specific exudate. The manifestations of the peritonitis in reactive and terminal phases are quite different. In the reactive phase, the pain syndrome prevails. The area of the utmost tenderness corresponds to the primary inflammatory focus. The rigidity of the abdomen, due to contraction of the muscles of the abdominal wall is a characteristic feature. The tense abdomen looks somehow tucked and boat-like. This sigh is of peculiar importance for the peritonitis with perforation of the hollow organs. Mendel and Schotkin-Blumberg symptoms are revealed. The peristalsis sounds are usually weakened or absent. The overt signs of dehydration are absent. The tachycardia may reach 90-100 beats per minute. The respiration rate is not increased. Several elements of the excitation phase of the shock such as the elevated blood pressure may also be evident. The
  • 12. 12 neutrophilic leukocytosis 12-18 х 109/L, lympho- and monopenia, aeosinophilia, may be present. The biochemical parameters are usually within the normal limits. In the terminal phase the diagnosis of the peritonitis may be rather difficult if the anamnestic details are not available. The vomiting with the congestive content takes place several times a day. The stool is scarce, frequently only after the enema. The stool has the putrefactive odor. The objective state: Adynamia, the general inhibition, the acute dehydration, the hollow eyes, the sharpened features of the face, the superficial frequent respiration, frequently with the quiet moaning. The tongue is dry and difficult to hang out; the internal buccal surfaces are also dry. The abdomen is inflated, weakly painful. Mendel and Schotkin-Blumberg symptoms are rather dubious. The percutory sound is not uniform. The symptoms of the deathly silence may be evident when the respiratory and the vascular noises are heard instead of the intestinal movements. The splash sound may be detected when the abdominal wall is stroke by the hand. The multiple Kloiber bowls are evident in survey X-ray pictures. The tachycardia is above 120 beats per minute. The blood pressure is decreased, especially after the transportation of the patient. The maintenance of the normal blood pressure requires the intensive infusion. The breathing rate is about 30 per minute. The oxygen saturation of the blood is diminished. The shadows of various sides in the lungs are characteristic of the respiratory distress syndrome. Hydrothorax may be developed. In blood, normocytosis, sometimes leukopenia are detected with the sharp shift to the left (towards the immature elements), sometimes plasma cells are detected. GENERAL PRINCIPLES OF TREATMENT 1. The early elimination of the infection source. 2. The decrease of the possible intraoperative contamination. 3. The treatment of the residual infection and the prevention of further infection of the abdominal cavity. 4. The maintenance of vitally important functions prior to and after the surgery, incuding the techniques requiring the artificial organs. METHOD OF TREATMENT А. Surgical Laparotomy, early elimination or isolation of the peritonitis source. Intra- and postoperative sanitization of the abdominal cavity. Decompression of the small intestine.
  • 13. 13 B. General Massive antibioticotherapy. Medicamentous correction of hemostasis shifts. Stimulation or temporary substitution of the major detoxifying systems of the body by means of extracorporeal hemocorrection. PREOPERATIVE PREPARATION. It is impossible to correct the hemostasis impairment prior to the surgery. Nevertheless, it is advantageous to delay the surgery for 2-3 hours for directed preparation. It is sufficient to stabilize arterial pressure and central venous pressure and to reach the diuresis at 25 mL/h. The total preoperative infusion volume is 1.5-2.0 L within two hours. In advanced cases when the hemodynamic imbalance is pronounced (the loss of fluid exceeding 10% of body mass), the infusion volume increases up to 3-4 L within 2-3 hours. The catheterization of the central (for example subclavian) vein is always performed providing for high infusion flow rate and the control of the central venous pressure. The catheterization of the urinary bladder is expedient providing for measuring the diuresis as the objective criterion of the infusion therapy efficacy. PREPARATION OF GASTROINTESTINAL TRACT. The stomach is emptied with the aid of the probe. In the advanced cases, the permanent presence of the probe in the stomach is expedient pre-operatively, intraoperatively and postoperatively for specified time until the recovery of gastric motor activity. ANESTHESIA. The multicomponent anesthesia with the artificial lung ventilation is used in surgery due to peritonitis. MAJOR OPERATIVE STAGES IN PERITONITIS 1. Surgical access. 2. Possible Novocain blockade of the reflexogenic areas. 3. Elimination or reliable isolation of septic focus. 4. Sanitization of the abdominal cavity. 5. Decompression of the intestine. 6. Draining of the abdominal cavity. 7. Suturing of the laparotomic wound. 1. Surgical access. The optimal access to all divisions of the abdominal cavity is provided by the midline laparotomy. Depending on the localization of the septic focus, the wound may be
  • 14. 14 extended in superior or inferior direction. When the diffuse peritonitis is revealed in the course of the surgery started from another access, it is worth to perform the midline laparotomy. 2. Novocain blockade of the reflexogenic areas is performed by administering 0.5% solution of Novocain (up to 100.0 mL) into the area of the celiac trunk, the root of the mesocolon, transverse mesocolon, the mesentery of the small intestine, and sigmoid mesocolon. Such blockade eliminates the reflex vascular spasm providing for earlier recovery of peristalsis. In addition, such blockade allows diminishing the amount of the narcotic substances used as the analgesics. 3. Elimination or reliable isolation of septic focus. In the reactive phase, the radical operations (resection of the stomach, hemicolectomy) are possible since the risk of the anastomosis failure is rather low. In the toxic and terminal phases the scope of the surgery should be minimal (appendectomy, the suturing of the perforative hole, the resection of the necrotic area of the gastrointestinal tract with entero- or colostoma, the isolation of the septic focus from the free abdominal cavity. The reconstructive surgery should be postponed until more favorable conditions. 4. Sanitization of the abdominal cavity. The washing of the abdominal cavity allows for decreasing the bacterial count in the exudates below the critical level (105 in 1 mL) facilitating the elimination of the infection. The dense deposits of fibrin are not removed because of the risk of deserozation. The removal of exudate by means of the wiping with the gauze pieces is unacceptable due to the risk of the injury of the serous membrane. The washing fluid must be isotonic. The use of antibiotics is meaningless because the short-term contact with the peritoneum is not sufficient for their effects. Most antiseptics are cytotoxic; this fact restricts their use in the cases under consideration. With this aim, one may use sodium chloride solution upon electrochemical activation (0.05% sodium hypochlorate) containing activated chlorine and oxygen. This solution is particularly indicated in then cases when anaerobic bacteria are present. Several clinics also use the ozonized solutions. 5. Decompression of the intestine. In the toxic and terminal phases when the intestinal paresis is of particular clinical importance, nasogastrointestinal intubation is performed. The polyvinylchloride tube is used with this aim. The intubation is extended distally from Treitz ligament. When necessary, the colon is intubated through the anus. Rarely, gastro-, jejuno- or appendicostoma is required for passing the probe. Postoperatively, the probe-assisted correction of the enteral content is performed comprising the decompression, the intestinal lavage, the enterosorption, and early enteral feeding. All these measures are useful for decreasing the permeability of the intestinal walls for the bacteria and the toxins facilitating early recovery of the functional activity of the gastrointestinal tract.
  • 15. 15 6. Draining of the abdominal cavity is performed by the tubes made of polyvinylchloride or rubber. Multichanneled polyvinylchloride drainages are the good choice for this procedure. One channel is used for supplying the antiseptic, while another one is used for the aspiration of the exudate. 7. Suturing of the laparotomic wound may be performed with the draining of the subcutaneous fat. The treatment of the residual infection, namely the techniques of the draining and the exudate removal, depends on the techniques used for the final stages of the surgery. Methods of suturing laparotomic wound: 1. The tight suturing without the drainage may be employed only in the case of the local, non-circumscribed peritonitis serous peritonitis with the low level of the bacterial load and low risk of abscesses and infiltrates. In these cases, the self-defense of the body and the use of antibiotics may be sufficient for curbing the infection. 2. The suturing with the passive drainages. The drainages are also used for the local administration of antibiotics. 3. The suturing with the drainages for the flow and fractional lavage. The technique is not employed in wide scale due to the difficulties in correcting the shifts in protein and electrolytic balance. Moreover, the efficacy of the technique lowers in 12-24 hours since the beginning of the lavage. 4. Semi-closed technique with approaching of the wound edges provides for the drainages being installed in the posterior abdominal wall for the dorsoventral washing with the aspiration of the outflowing fluid via the midline wound. 5. The approaching of the wound edges with the specific appliances used for the repeated revisions and sanitizations. We use the term "scheduled laparosanitization". The indication for such technique is the pronounced adhesion process in the severe forms of the fibrinopurulent peritonitis with sub- and decompensation of the vitally important functions. The number of the revisions is from 2-3 to 7-8 with 1248-hours interval. 6. The open technique (laparostomy after N. Macokh or Steinberg-Miculich) with the exudate outflow through the wound covered by the tampons with the ointment is used in the case of the multiple non-shaped intestinal fistulas. The doctor may observe the condition of the intestinal loops adjacent to the wound in every change of the tampons.
  • 16. 16 GENERAL TREATMENT Antibacterial therapy The most adequate schedule of the empirical antibacterial therapy before the microbiological verification of the causative agent and the assessment of its sensitivity to antibiotics is a combination of the synthetic penicillins (Ampicillin) or cephalosporins with aminoglycosid (Gentamicin or Vancomicin) and Metronidasol. Such a combination seems to be effective against the broad spectrum of the possible causative agents of the peritonitis. When the data of bacteriological analysis are obtained, the corresponding antibiotics or their combination are prescribed. The routes of the administration: 1) Local (intraperitoneal) through irrigators and drainages (the double function of draining) 2) General a) intravenous (systemic) b) intramuscular (only after the recovery of microcirculation) c) intra-arterial (into aorta, celiac trunk, mesenteric or omental artery) d) intraportal (through the recanalized umbilical vein of the round ligament of the liver (regional way) e) endolymphatic – antegradely: through microsurgically catheterized peripheral lymphastic vessel in the back of the foot, the depulpated inguinal lymph node – retrogradely: through thoracic lymphatic duct – lymphotropically: through the lymphatic net of the shin or retroperitoneal space. Immunotherapy The following preparations improving the immune reactivity of the body may be used: immunoglobulins (antistaphylococcus gamma-globulin), the leukocytic mass, antistaphylococcus plasma. Interleukin-2, α- and γ-interferons, interferon inducers (cycloferon) also may be employed as the modern drugs improving the immune reactivity. Meanwhile, the use of Pyrogenal, Decaris (Levamisol), Prodigiosan, Thymalin and other agents "stimulating the reduced immunity" seems to de contraindicated.
  • 17. 17 Postoperative correction therapy For the adequate anesthetization, besides the narcotic analgesics, the prolonged epidural analgesia with the local analgesics, acupuncture analgesia, and electroanalgesia are employed. The balanced infusion therapy is performed. The total daily volume of the infused fluid comprises the physiological daily requirement (1500 mL/m2), the deficit of the liquid, and the physiological expenses (vomiting, drainage, increased sweating, and hyperventilation). The preventing and the treatment of multiple organ failure syndrome: The pathogenetic bases of the multiple organ failure syndrome is hypoxia and cell hypotrophy due to the impaired respiration and hemodynamics. The following measures should be undertaken to prevent and to treat multiple organ failure syndrome: – Elimination of the infectious and toxic focus. – Elimination of the toxins by the techniques of the efferent therapy. – Maintenance of the adequate lung ventilation and gas exchange (sometimes long=-term artificial lung ventilation). – Stabilization of blood circulation with restoration of the blood circulation volume, the support of heart function. Normalization of microcirculation in organs and tissues. – Correction of protein, electrolyte, and acid-base balance of the blood. – Parenteral feeding Recovery of functions of gastrointestinal tract The most effective means for recovering the functions of gastrointestinal tract consists in the intestinal decompression by transnasal probe followed by the intestinal lavage. For the normalization of the nervous regulation and the recovery of the tonus of intestinal musculature, the correction of the protein and electrolyte balance is performed. Then anticholinesterase drugs (Proserin, Ubretid), Metaclopramide and ganglioblockers (Dimecolin, Benzohexonium) may be used. The forced diuresis, hemodialysis, plasmapheresis, hemofiltration through the porcine organs, the artificial lung ventilation, hyperbaric oxygenation are indicated in case of the multiple organ failure syndrome. The hyperbaric oxygenation is capable for arresting all types of hypoxia in peritonitis, facilitates the decrease in the bacterial load of the peritoneum, stimulates motor and evacuatory function of the intestine. For preventing endotoxicemia, the surgical methods and antibacterial therapy are to be used for eliminating the residual infection. Hemosorption, lymphosorption, plasmapheresis and other detoxifying techniques are to be regarded only as the supplementary methods in the complex therapy of peritonitis.
  • 18. 18 VI. Plan and arrangement of training 6.1 Duration of classes: 4 academic hours 6.2. Stages of classes: N Content and program of Educa- Methods of Supporting Time teaching tional teaching and materials in objectives control min within learning levels 1 2 3 4 5 6 І Preparatory step Log-book for 1 Organization of classes entering the 5 2 Formulation of the teaching learning progress goals of students 3 Preparatory stage (control of the initial level of knowledge Tables, slides 20 and skills): α-ІІ Tests α-ІІ Tables, slides а) major concepts on the 20 anatomy and physiology of α-ІІ Tests α-ІІ Tables, slides peritoneum; α-ІІ Tests α-ІІ 15 б) infectious process; Typical check в) approaches for the surgical situations treatment; ІІ Principal step Equipment and 120 1. Training the professional facilities skills and knowledge Training 2. Examination of the patients α-ІІІ with peritonitis Non-typical 3. Differential; diagnosis of check peritonitis. Diagnostical and situations clinical features. α-ІІІ ІІІ Final step Individual Control questions , 1. Control and correction of the control of tasks. 50 attained level of professional α-ІІ students' Advice for skills knowledge homework with the 5 2. Summary of classes literature (theoretical, practical, α-ІІІ organizational) 5 3. Home tasks VII Supporting materials required for teaching Materials for the control of initial level of knowledge and skills: А. Control questions: 1. Anatomical relations between the peritoneum and the organs of the abdominal cavity 2. Histological structure and innervation of the peritoneum.
  • 19. 19 3. Physiology of the peritoneum: resorptive and exudative functions. 4. Definition of peritonitis. 5. Classification of peritonitis according to the clinical course. 6. Classification of peritonitis according to the character of penetration of bacteria into the abdominal cavity. 7. Infectious inflammatory peritonitis. 8. Perforative peritonitis. 9. Traumatic peritonitis. 10. Postoperative peritonitis. 11. Classification of peritonitis according to the microbiological features. 12. Bacterial peritonitis. 13. Aseptic peritonitis. 14. Specific forms of peritonitis. 15. Classification of peritonitis according to the character of the peritoneal exudate. 16. Classification of peritonitis according to the character of lesions on the surface of the peritoneum. 17. Circumscribed peritonitis. 18. Non-circumscribed peritonitis. 19. Classification of peritonitis according to the extent of spreading. 20. Localized peritonitis. 21. Generalized (diffuse) peritonitis. 22. General (total) peritonitis. 23. Phases of the development of peritonitis. 24. Reactive phase of peritonitis. 25. Toxic phase of peritonitis. 26. Terminal phase of peritonitis. 27. Complications of peritonitis. 28. Microbiological features of peritonitis. 29. Patients' complaints and the anamnestic data in peritonitis. 30. General and specialized examination of the patients with peritonitis. 31. Laboratory examinations 32. Supplementary examinations. 33. General principles of treating peritonitis. 34. Method for treating peritonitis. 35. Preoperative preparation. 36. Preparation of gastrointestinal tract. 37. Anesthesia. 38. The major steps of the surgical treatment of peritonitis. 39. Draining of the abdominal cavity and suturing of the laparotomic wound in peritonitis. 40. The general treatment of peritonitis. TESTS І. The most pronounced exudative activity is peculiar to: A. peritoneum of ileum; B. peritoneum of duodenum; C. peritoneum is devoid of exudative activity; D. peritoneum of diaphragm; E. peritoneum of sigmoid colon.
  • 20. 20 ІІ. The most pronounced resorptive activity is peculiar to: A. peritoneum of jejunum; B. peritoneum of duodenum; C. peritoneum is devoid of resorptive activity; D. peritoneum of diaphragm and greater omentum; E. peritoneum of sigmoid colon. ІІІ. The peritoneum covers (completely or partially) all the organs of the abdominal cavity except for: A. pancreas; B. kidneys; C. urinary bladder; D. omentum; E. in the abdominal cavity all the organs without exception are covered by the peritoneum. IV. Select the wrong statement – peritonitis may be: A. primary; B. secondary; C. perforative; D. traumatic; E. all the statements are correct. V. Infectious and inflammatory peritonitis is not a consequence of: A. acute appendicitis; B. acute cholecystitis; C. acute ileus; D. traumatic injuries of parenchymatous organs; E. gynecological pathology. VI. Which of the form in the list below is not regarded as the specific form of peritonitis: A. infectious; B. parasitic; C. rheumatoid; D. granulomatous; E. carcinomatous. VII. Which of the pathologies from the list below do not belong to the systemic complications of peritonitis: A. pylephlebitis; B. syndrome of the disseminated intravascular blood coagulation; C. toxic encephalopathy; D. renal failure; E. cardiovascular failure. VIII. Select the wrong statement – the following symptom does not belong to the pathognomonic symptoms of peritonitis: A. Mussi-Heorhievskyi symptom; B. Ortner symptom; C. Karavaiev symptom; D. Schotkin-Blumberg symptom; E. Pasternatsky symptom.
  • 21. 21 IX. The stage which does not comprises the major stages of the surgical treatment of peritonitis: A. operative access; B. elimination or reliable isolation of the septic focus; C. sanitization of the abdominal cavity; D. draining of the abdominal cavity; E. antibioticotherapy. Х. Select the wrong statement: Paresis of the intestine… A. ...accompanies the development of peritonitis; B. …is a pathogenetic factor contributing to the severity of the clinical course in peritonitis; C. …in peritonitis is subjected to the conservative correction; D. … is not observed in all cases of the local form of peritonitis; E. …is in most cases an indication for intestinal intubation. The correct answers: 1–A 4–E 7–A 10 – C 2–D 5–E 8–A 3–E 6–D 9–D REFERENCES 1. S.V. Petrov. «General surgery». Peter. 2005 (in Russian). 4. Yu.M. Lopukhin, V.S. Saveliev «Surgery» Moscow. 1997 (in Russian). 5. C.M. Townsend. Textbook of Surgery. The biological basis of modern surgical practice. 2001.